Burns and Burn Reconstruction 01-22 Flashcards

1
Q

A 19-year-old woman presents with a 2 × 3-cm burn to the left dorsal thumb sustained from brief exposure to hot bacon grease. During the next 24 hours, the burned area becomes erythematous and several blisters containing clear fluid have formed and then ruptured. Examination shows no erythema extending proximally. A photograph is shown. Which of the following is the most appropriate next step in management?

A) Dry dressing
B) Hydrogen peroxide soaks
C) Leave open to air
D) Topical antibiotic ointment

A

The correct response is Option D.

Based on the information provided, this is likely a superficial partial-thickness burn. It is not deep enough or large enough to warrant transfer to a burn center. Local wound care should lead to full resolution, with appropriate follow-up to detect delays in healing. Partial-thickness wounds like this will heal fastest when a moist environment is maintained. Antibiotic ointment or other neutral moisturizing agents (e.g., petrolatum ointment) are appropriate for local wound care. Leaving the wound open to air or covering with a dry dressing will delay healing. Hydrogen peroxide is appropriate for cleansing dirty or infected wounds, but it is not necessary in the scenario presented. Furthermore, sustained use of hydrogen peroxide may delay healing in clean wounds.

How well did you know this?
1
Not at all
2
3
4
5
Perfectly
2
Q

An otherwise healthy 20-year-old man is brought to the emergency department after sustaining burns to the right forearm and upper arm while trying to light an outdoor grill. Examination of the burned extremity shows red, blistered, painful areas, as well as areas that are white in color and not tender to palpation. The burns are noncircumferential. The patient is breathing comfortably, and pulse oximetry is 99% on room air. This patient should be transferred to a burn center on the basis of which of the following criteria?

A) Age of the patient
B) Burn depth
C) Burn location
D) Burn mechanism
E) Total body surface area of the burn

A

The correct response is Option B.

The depth of this patient’s burns, being third-degree (white, not tender to palpation), would qualify him for transfer to a burn center for treatment. In this patient, the arm represents a total body surface area of 9% of the patient. He is young and otherwise healthy; thus, he does not warrant transfer based on age. The burn does not involve sensitive areas such as the face, hands, feet, or genitalia/perineum. Also, since this is not an electrical or chemical burn, and it occurred in an outdoor area, inhalation injury is unlikely.

How well did you know this?
1
Not at all
2
3
4
5
Perfectly
3
Q

A 29-year-old man presents with deep second- and third-degree burns on 35% of total body surface area. Fluid resuscitation via the Parkland formula is performed. Which of the following parameters is most commonly used to assess for adequacy of resuscitation?

A) Blood pressure
B) Heart rate
C) Pulmonary wedge pressure
D) Pulse oximetry
E) Urine output

A

The correct response is Option E.

Patients who sustain greater than 20% total body surface area second- or third-degree burns will require aggressive resuscitation to prevent development of burn shock. While the Parkland formula and other such resuscitation protocols are useful for initiation of proper burn resuscitation, these measures should only be used as a starting point, with the resuscitation being guided by the physiologic response of the patient. The most important parameter to assess the adequacy of resuscitation is urine output. A goal for adequate resuscitation efforts should be above 0.5 mL/kg/hr for adults and closer to 1 mL/kg/hr for children. It is important to note that over-resuscitation can also result in what has been recently described as “fluid creep,” and includes significant complications such as excessive edema, third spacing, and even abdominal compartment syndrome, so fluid infusions should also be lowered if urine output is significantly higher than these rates. While monitoring of other physiologic parameters such as blood pressure, heart rate, and pulse oximetry are important, and serial measurements of cardiac output are increasingly being used, they are not more commonly used than urine output to assess the adequacy of resuscitation or help to prevent over-resuscitation. Pulmonary wedge pressure is no longer a common modality for monitoring fluid resuscitation status.

How well did you know this?
1
Not at all
2
3
4
5
Perfectly
4
Q

A 38-year-old electrician suffers an electrical burn to the right hand. Which of the following types of tissue has the lowest inherent resistance?

A) Bone
B) Fat
C) Muscle
D) Skin
E) Tendon

A

The correct response is Option C.

The tissue with the least resistance from the choices provided is muscle. The amount of electrical current that is conducted through tissue is proportional to the voltage and inversely proportional to the tissue’s resistance, as is dictated by Ohm’s Law, V=IR. Therefore, more current will flow through tissue with lower resistance, given all other variables being equal. Other variables that effect current flow include tissue volume and the amount of moisture. The body tissue with the highest inherent resistance is cortical bone, followed by cancellous bone, fat, tendon, skin, muscle, vessel, then nerve. So, in the setting of a high-voltage electrical burn, it is not uncommon to have more underlying structural and organ damage than the visible soft tissue damage may indicate. An EKG, cardiac monitoring, CBC, cardiac enzymes, and urinalysis for myoglobin may, therefore, be necessary for workup.

How well did you know this?
1
Not at all
2
3
4
5
Perfectly
5
Q

A 30-year-old man sustained a third-degree burn to his right arm from a flame while cooking over a grill. The burn is 5% total body surface area (TBSA). Two days after the injury, he undergoes debridement of the dorsal wrist and forearm. The paratenon is not present after the debridement. A bilaminate neodermis (Integra) graft is selected and placed on the wound. Use of this graft is associated with which of the following?

A) Decreased cost to the hospital
B) Decreased number of hospital stays
C) Decreased number of surgeries
D) Decreased risk for hypertophic scar
E) Increased skin sensation after reconstruction

A

The correct response is Option D.

The literature states that there is a decrease in hypertrophic scarring associated with the use of bilaminate neodermis (Integra) with burn reconstruction. The cost of the product is high. Use of the product requires a second surgery for the skin graft. There has not been any literature supporting improved sensation following use of the graft. The downside to the graft is that it can result in longer hospital stays for the patient in order to get the second surgery completed.

How well did you know this?
1
Not at all
2
3
4
5
Perfectly
6
Q

A 19-year-old man reports severe, worsening arm pain and finger swelling accompanied by distal numbness. He sustained a circumferential burn to the arm 12 hours ago. The area of burned skin itself is noted to be leathery and insensate. Examination demonstrates loss of distal pulses. Which of the following is the most appropriate next step?

A) Angiography
B) CT scanning
C) Duplex ultrasonography
D) Escharotomy
E) Fasciotomy

A

The correct response is Option D.

The most appropriate next step is escharotomy.

The patient exhibits signs of vascular compromise due to tight restrictive burn eschar arising from a circumferential full-thickness burn. Edema following a burn due to inflammation and fluid resuscitation can cause increased swelling of the tissues, and the presence of tight circumferential eschar can give rise to vascular compromise. Release of the burn eschar (escharotomy) is indicated to relieve pressure on the tissues and allow for restoration of blood flow.

Burns are classified into partial thickness and full thickness, and into different degrees based on the depth of injury. In the case of full-thickness (third-degree or fourth-degree) burns, the area of burned skin is insensate and may appear charred or leathery. Eschar formation in circumferential burns can lead to a tourniquet effect, with impaired circulation. In some cases, circumferential burns of the torso may even give rise to respiratory compromise or abdominal compartment syndrome.

Escharotomy differs from fasciotomy in that the incision is made more superficially, to open the thick burn eschar, and does not need to extend deep to the fascia.

Angiography would be useful in evaluating vasculature and blood flow, but would not be needed in this situation and would delay treatment.

CT scanning can provide detailed imaging, but it would not be indicated in this situation and would delay treatment.

Duplex ultrasonography can evaluate the presence of deep vein thrombosis, which could cause pain and swelling. However, in this case, the clinical scenario suggests that circulatory restriction is due to the circumferential burn scar.

Fasciotomy is recommended in the case of increased compartment pressures, which may also present with pain, paresthesia, and loss of pulses, although in this instance the presence of a circumferential burn would indicate escharotomy as the treatment.

Fasciotomy is advised if compartment pressure exceeds 30 mmHg, or if the difference between intracompartmental pressure and diastolic blood pressure is less than 30 mmHg.

How well did you know this?
1
Not at all
2
3
4
5
Perfectly
7
Q

A 50-year-old woman with a history of hypertension is brought to the hospital 1.5 hours after sustaining burn injuries in a house fire. Initial examination shows deep, second-degree burns (partial-thickness) to 35% of the trunk, non-circumferential third-degree burns (full-thickness) to 5% of the left forearm, and first-degree burns to 2% of the head. In addition to time from injury, which of the following common aspects of the burn evaluation should be used in both the Parkland formula and Brooke formula for determining fluid resuscitation in this patient?

A) Total body surface area (TBSA) (determined by first-, second-, and third-degree burns), gender, weight
B) TBSA (determined by first-, second-, and third-degree burns), weight
C) TBSA (determined by second- and third-degree burns), age, weight
D) TBSA (determined by second- and third-degree burns), gender, weight
E) TBSA (determined by second- and third-degree burns), weight

A

The correct response is Option E.

While there are different resuscitation formulas for initial burn resuscitation, such as the Brooke formula or the Parkland formula, they rely on giving a certain amount of fluid multiplied by total body surface area (as determined by partial- and full-thickness burns) and weight in kilograms of the patient. The fluid is then given initially as determined from time of injury, and divided into half given in the first 8 hours of injury and then half in the next 16 hours. This is a guideline only and resuscitation can be altered based on physiologic response, such as urine output. Gender and age are not a consideration and first-degree burns are not used in the calculation of total body surface area.

How well did you know this?
1
Not at all
2
3
4
5
Perfectly
8
Q

An otherwise healthy, 32-year-old woman with a history of burns on 20% of the total body surface area of the left upper extremity and chest, returns 1 year after her injury for evaluation of a nonhealing wound over the dorsal elbow and limited range of motion at the joint. X-ray studies show soft-tissue lamellar calcification. Which of the following is the most likely diagnosis?

A) Chronic osteomyelitis
B) Heterotopic ossification
C) Hypertrophic scar
D) Marjolin ulcer
E) Retained foreign body

A

The correct response is Option B.

The most likely diagnosis is heterotopic ossification (HO). An important complication of massive burn injury (greater than 20 % BSA) is heterotopic ossification, with the elbow region being the most common site of occurrence. This may result in wound issues, stiffness, and nerve injury. HO can occur in 0.2 to 4 % of cases. Prevention of this condition includes radiation therapy and nonsteroidal anti-inflammatory drugs. Surgical excision is the procedure of choice for restoration of range of motion.

Hypertrophic scars present as raised thickened scars following burn injury but do not show ossification within the scar. The most common scenario for development of a Marjolin’s ulcer is malignant degeneration of a previous scar, usually squamous cell carcinoma. The latency period for development of this type of malignancy is usually 10 years after the original injury at the earliest. Chronic osteomyelitis would show radiolucency and changes within the bone not the soft tissue. The x-ray finding is not consistent with a retained foreign body.

How well did you know this?
1
Not at all
2
3
4
5
Perfectly
9
Q

A 24-year-old man presents to the emergency department after sustaining burns from a workplace fire in a kitchen. He has a suspected inhalation injury. Physical examination shows partial-thickness burns on the forearms and thighs, as well as on one ear. The plan is to use mafenide acetate as a topical antimicrobial dressing. Because of the suspected lung injury, mafenide should be used with caution because it places him at increased risk for which of the following?

A) Gray discoloration of skin
B) Hyperosmolality
C) Metabolic acidosis
D) Methemoglobinemia
E) Pruritis

A

The correct response is Option C.

The topical use of mafenide acetate, a carbonic anhydrase inhibitor, can prevent the conversion of hydrogen ions in the body to carbonic acid, leading to metabolic acidosis. In patients with an inhalation injury and respiratory acidosis, the use of mafenide acetate over large surface areas can be fatal.

The use of mafenide is common in areas with cartilage, such as the ear. It is also useful for eschar penetration. Common adverse effects include pain with application. Silver sulfadiazine is associated with leukopenia and hyperosmolality. Silver nitrate can also be used, but it causes gray-to-black staining of the wound and can also be associated with electrolyte imbalances. Methemoglobinemia has also been reported with silver nitrate use.

How well did you know this?
1
Not at all
2
3
4
5
Perfectly
10
Q

An otherwise healthy, 76-year-old woman sustains a deep partial-thickness facial burn following scald injury. No acute intervention is performed and the patient follows up in the clinic 6 weeks after the incident. She complains of pain in the right eye as well as frequent tearing. Physical examination of the eye shows 4 mm of lagophthalmos, conjunctival injection, and hypertrophic scarring on the upper lid with restrained motion. The surrounding skin on the cheek and forehead appears erythematous and indurated. Which of the following is the most appropriate treatment for this patient?

A) Contracture release and forehead flap
B) Contracture release and full-thickness skin grafting
C) Contracture release and placement of allograft
D) Contracture release and split-thickness skin grafting
E) Scar massage and eyelid taping

A

The correct response is Option B.

Facial burn injuries in the periorbital area must be quickly treated to support patient comfort and protect vision. Continued lagophthalmos and patient complaints warrant intervention, particularly with the degree of symptoms that the patient is expressing. The need for earlier intervention is specifically discussed in Klifto, et al. There is no indication for allograft placement following contracture release in this case, as a surface to graft upon should be present following contracture release in the eyelid. Of note, the contracture release usually occurs superficial to the orbicularis oculi and superior to the tarsal plate. Following contracture release, full-thickness skin grafts, but not split-thickness skin grafts, will minimize contracture after placement. Forehead flap reconstruction of the upper eyelid is a reasonable intervention and may be the most successful in preventing contracture recurrence, but it is not a treatment choice if the donor skin may be involved with the burn injury, as in the case of this patient.

How well did you know this?
1
Not at all
2
3
4
5
Perfectly
11
Q

A 56-year-old man who works at a fertilizer production plant presents to the emergency department with 10% hydrofluoric acid burns to the palmar surface of both hands. He is in exquisite pain. Physical examination shows no other injuries. Which of the following is the most appropriate management of this burn injury?

A) Application of a dilute alkali to neutralize the acid
B) Copious surface irrigation and application of calcium gluconate gel
C) Frequent electrolyte and renal lab analyses due to systemic toxicity
D) Mafenide acetate (Sulfamylon) dressings
E) Pain control and maintenance intravenous fluids

A

The correct response is Option B.

The patient has sustained a hydrofluoric acid burn, an agent used in many industrial and domestic applications. Hydrofluoric acid is a unique acidic chemical agent because it can behave as a strong acid at higher concentrations and can also cause liquefactive necrosis, as alkalis do by the dissociation of fluoride ions into subcutaneous tissues. Fluoride ions combine with calcium resulting in local hyperkalemia, which is believed to be the cause for the “pain out of proportion” examination finding associated with hydrofluoric acid burns.

Application of a dilute alkali is not recommended for the treatment of acidic burns because the resulting reaction can be exothermic, resulting in additional injury as the acid is being neutralized.

For patients with greater than 5% total body surface area (TBSA) exposure to hydrofluoric acid or injury with less than 50% concentration of hydrofluoric acid, systemic toxicity can result, causing electrolyte disturbances and organ dysfunction. This patient’s injury resulted from a low-concentration hydrofluoric acid exposure to a small surface area, making serial laboratory assessment less of a priority during presentation.

Due to the intense pain that hydrofluoric acid burns can cause, pain control will be required for management, but maintenance fluid and pain control alone will not treat the burn injury.

The hallmark of hydrofluoric acid chemical burn injury treatment is skin surface irrigation with copious amounts of water at lower pressure, followed by topical calcium gluconate to bind the fluoride ions before they penetrate into the soft tissues. This will neutralize the burn reaction, bind the fluoride ions, and help with pain control.

Mafenide acetate is an appropriate topical antiseptic for non-chemical burns, but will not neutralize hydrofluoric acid.

How well did you know this?
1
Not at all
2
3
4
5
Perfectly
12
Q

A 66-year-old, 132-lb (60-kg) woman presents to the emergency department with a deep second-degree, 20% total body surface area burn, with a small area of surrounding first-degree burn. This happened in an open space when she fell backwards onto a fire pit. Her burns are isolated to her buttock and back. She reports no hoarseness or difficulty breathing. She receives 9600 mL of Ringer’s lactate within the first 24 hours. Based on her fluid resuscitation, she is at highest risk for which of the following?

A) Abdominal compartment syndrome
B) Conversion of the burn to full-thickness
C) Deep venous thrombosis
D) Digit ischemia
E) Poor engraftment of autologous skin grafts

A

The correct response is Option A.

Numerous formulas regarding burn resuscitation have been developed to avoid under-resuscitation. This was based on previous literature suggesting that under-resuscitation was associated with significant end organ damage secondary to ischemic injury. Inadequate resuscitation was similarly associated with the potential for hemodynamic collapse, resulting in death. Weight-based resuscitation programs and establishment of urine output guidelines have largely limited under-resuscitation at burn centers. However, burn patients now suffer from the consequences of over-resuscitation, in which patients receive even more fluid than recommended by the Parkland formula. These patients suffer from increased rate of burn infections, the development of acute respiratory distress syndrome, and abdominal compartment syndrome.

In the clinical case presented, the Parkland formula would suggest that the patient receive approximately 4800 mL of resuscitation (4 × TBSA burn [20] × weight in kilograms [60]) within the first 24 hours. This patient received approximately double the amount.

Regarding engraftment, there is no evidence that over-resuscitation worsens autologous skin graft engraftment. Conversion of burn injuries to deeper injuries is usually associated with under-resuscitation, as is end organ or digit ischemia. Deep venous thrombosis does not have any reported correlation with burn resuscitation.

How well did you know this?
1
Not at all
2
3
4
5
Perfectly
13
Q

Which of the following is associated with the use of pressure garments in the management of burn scars?

A) Decreased scar strength
B) Increased synthesis of tissue proteinases
C) Larger and less densely packed collagen fibers
D) Reduced differentiation of fibroblasts to myofibroblasts

A

The correct response is Option D.

Significant differences in scar contraction were observed between scars receiving pressure garment therapy and control burns that received no pressure. Pressure garments exert compressive forces perpendicular and parallel to the surface of the scar. These forces oppose the direction of contracture. One hypothesis is that wound tension acts upon integrins by stretching them, which leads to phosphorylation of focal adhesion kinase and upregulation of smooth muscle actin and collagen production. When compression is applied to incisional wounds perpendicular to the wound tension, scarring is minimized. This suggests that the mechanical forces applied to the scar can assist in reducing differentiation of fibroblasts to myofibroblasts, decreasing scar contraction and collagen deposition.

Scar strength was improved with pressure garment therapy compared with controls, with a 34% increase in ultimate tensile strength. Pressure garment therapy scars were also found to be composed of smaller, more densely packed collagen fibers.

Increased synthesis of tissue proteinases is a mechanism of corticosteroids.

How well did you know this?
1
Not at all
2
3
4
5
Perfectly
14
Q

An 8-year-old girl presents with burns on 60% of the total body surface area. Physical examination shows second- and third-degree burns involving her face, neck, and torso. After initial resuscitation, wound care, burn debridement, and skin grafting are performed, rehabilitation protocol is initiated. Administration of which of the following agents is most appropriate for improving bone mineral content (BMC) in this patient?

A) Ascorbic acid
B) Glutamine
C) Insulin
D) Oxandrolone
E) Testosterone

A

The correct response is Option D.

A randomized clinical trial of safety and efficacy of 1-year oxandrolone administration to severely burned children (over 30% total body surface area burns) demonstrated significant benefits of this medication. Improvements were noted in height, bone mineral content (BMC), cardiac work, and muscle strength, and were statistically higher compared to the control group. Mechanism of action is not totally clear but increase in insulin-like growth factor-1 secretion during the first year after burn injury, and, in combination with exercise, considerable increase in lean body mass and muscle strength has been demonstrated. The maximal effect of oxandrolone was found in children aged 7 to 18 years. No deleterious side effects were attributed to long-term administration.

Oxandrolone, a synthetic oral nonaromatizable testosterone derivative, has only 5% of the virilizing activity and low hepatotoxicity when compared with testosterone administration. Oxandrolone reaches peak serum concentrations within 1 hour and is excreted through the urine. Oxandrolone binds to androgen receptors in the skeletal muscle to initiate protein synthesis and anabolism. Because oxandrolone cannot be aromatized to estrogen, the likelihood of estrogen-dependent bone-age advancement is reduced, making oxandrolone a safe therapeutic approach for growing children.

Testosterone is not currently approved for treatment of burned children due to increased risks of virilization in female patients and aromatization effects among other health risks.

Glutamine and ascorbic acid supplementation can aid in burn recovery; however, it has not shown to have similar effects on bone density as oxandrolone. Several studies support the use of enteral glutamine supplements in the adult burn population. Research has also shown that glutamine supplementation is favorable as it has the potential to decrease length of stay and associated costs through improving wound healing and decreasing rates of infection and mortality.

Antioxidant therapies including: ascorbic acid; glutathione; N-acetyl-L-cysteine; vitamins A, C, and E; alone or in combination have been previously shown to protect microvascular circulation, mitigate changes in cellular energetics, decrease tissue lipid peroxidation, and decrease the volume of fluid required for resuscitation.

Insulin is used to treat hyperglycemia and primarily used in diabetic patients. It may have limited use in burn care patients but has not shown to increase bone mineral density.

How well did you know this?
1
Not at all
2
3
4
5
Perfectly
15
Q

A 25-year-old man presents with partial-thickness burns involving 15% of the total body surface area that he sustained during a house fire. The patient is stabilized and resuscitated. Topical 1% silver sulfadiazine cream is applied to the burns. Which of the following properties is most characteristic of this antimicrobial agent?

A) Greatly enhanced efficacy when compounded with thiol chelators
B) Metabolic acidosis
C) Poor capacity for wound bed penetration
D) Poor efficacy against Candida albicans
E) Potential for transient leukocytosis

A

The correct response is Option C.

In the United States, silver is the most commonly used topical antimicrobial. It is available as a liquid solution of AgNO3 or ointments such as silver sulfadiazine (Silvadene). Despite its many advantages, its capacity to penetrate into the wound bed is limited to the surface epithelium, particularly in the presence of eschar because of the binding of silver ions to surface proteins. In this setting, different modalities should be used for optimal effects.

Silver sulfadiazine is not only effective against Pseudomonas species and enteric bacteria, but it also provides coverage against fungi, including Candida albicans, with antimicrobial effects lasting up to 24 hours.

Enhanced efficacy when compounded with thiol chelators is consistent with bismuth compounds, not silver. Bismuth is another heavy metal with antimicrobial properties.

The most commonly used formulation of bismuth for wound care is bismuth subgalactate, found in xeroform (Covidien) gauze. This heavy metal disrupts biofilm formation by inhibiting polysaccharide capsule production in bacteria. Bismuth’s antibacterial activity is enhanced when compounded with thiol chelators.

Regarding the potential for transient leukocytosis, silver sulfadiazine has been shown to cause reversible neutropenia, which usually improves within a few days after discontinuation of the agent.

Metabolic acidosis is associated with mafenide acetate use.

How well did you know this?
1
Not at all
2
3
4
5
Perfectly
16
Q

A 51-year-old farmer is brought to the emergency department after sustaining extensive burns in a fertilizer explosion. Examination shows white phosphorus embedded in his burn wounds. In addition to burn resuscitation and examination of the wounds under ultraviolet light, application of which of the following is the most appropriate next step in management?

A) Calcium gluconate
B) Mafenide (Sulfamylon)
C) Mineral oil
D) Polyethylene glycol
E) Saline irrigation

A

The correct response is Option E.

White phosphorus is sustained in both military and civilian circumstances. It is commonly found in fireworks, fertilizers, and pesticide. It is extremely volatile and can ignite spontaneously upon exposure to air. Additionally, phosphoric acids form during combustion and further injure tissues.

Treatment mainstays include:

Immediate debridement of visible debris

Copious irrigation

Keep the area wet and covered with saline-soaked gauze

Cardiac monitoring and electrolyte evaluation.

Profound hypocalcemia, hyperphosphatemia, and sudden death have been associated with this injury.

Calcium gluconate gel is used in the management of hydrofluoric acid burns. Polyethylene glycol is used in the management of phenol and cresol burns. Mineral oil is used to isolate potassium, sodium, and magnesium from water, with which they react explosively. Mafenide (Sulfamylon) has no role in the immediate management of white phosphorus burns.

How well did you know this?
1
Not at all
2
3
4
5
Perfectly
17
Q

A 25-year-old right-hand–dominant woman sustains a full-thickness circumferential burn to the right upper extremity from the shoulder to the wrist. She undergoes early excision and grafting. Six months after treatment, she undergoes operative release of a severe flexion contracture of the elbow (greater than 50% loss of joint motion), resulting in a large defect. Which of the following is the most appropriate option for reconstruction of the defect?

A) Free fasciocutaneous flap
B) Full-thickness skin grafting
C) Local perforator flap
D) Split-thickness skin grafting
E) Z-plasty

A

The correct response is Option A.

In severe burn scar contractures, adjacent tissue transfer (Z-plasty, VY-plasty) and skin grafts are not indicated. Perforator-based local flaps have low recurrence rates but one limitation of this technique is the availability of local normal skin. In this specific case, no normal skin is available. Free tissue transfer is the best option. Perforator vessels are normally protected and can serve as recipient vessels for the free flap transfer.

How well did you know this?
1
Not at all
2
3
4
5
Perfectly
18
Q

A 32-year-old man is brought to the emergency department with a full-thickness thermal burn injury to the left ear. Which of the following therapeutic agents allows for the most effective preservation of involved cartilage?

A) Acetic acid
B) Honey
C) Mafenide acetate
D) Nanocrystalline silver dressings
E) Silver sulfadiazine

A

Mafenide acetate effectively penetrates burn eschar as well as cartilage and decreases the risk of suppurative chondritis in the setting of burns of the auricle. Twice-daily application is recommended. Care must be taken to monitor for metabolic acidosis, as mafenide acetate is metabolized to sulfamoylbenzoic acid, a carbonic anhydrase inhibitor. Silver sulfadiazine, nanocrystalline silver dressings, honey, and acetic acid have not demonstrated similar efficacy in preventing burn-associated chondritis.

How well did you know this?
1
Not at all
2
3
4
5
Perfectly
19
Q

A 43-year-old woman comes to the emergency department 2 hours after sustaining deep second-degree burns to the right elbow from a campfire. The wound measures 8 x 20 cm. After surgical excision, placement of autografts, and coverage with petrolatum gauze, which of the following dressings is most likely to promote graft survival?

A) Cotton balls and tie-over bolster dressing

B) Dry gauze and compressive wrap

C) Negative pressure wound therapy

D) Occlusive dressing

E) No additional dressing

A

The correct response is Option C.

In the patient described, the most effective way to fixate autografts is negative pressure wound therapy (NPWT). Several studies in burn patients have demonstrated the superiority of NPWT over the other conventional dressings listed. Whether staples or sutures are used has little effect on graft survival, although sutures may offer more precise graft placement. Most dressings for graft fixation use petrolatum gauze of some form, followed by some type of dressing that offers compression to fixate grafts. Dry gauze and a compressive wrap is the simplest option. Cotton balls with a tie-over bolster dressing are also effective, especially for small grafts. Larger surface areas are more difficult to secure with this method. A simple occlusive dressing, while acceptable for a donor site, will not effectively fixate autografts. NPWT offers good graft fixation, exudate removal, and promotion of local perfusion, which may explain the improved graft survival observed.

How well did you know this?
1
Not at all
2
3
4
5
Perfectly
20
Q

A 45-year-old man is brought to the burn unit after sustaining injuries during a house fire. Heart rate is 112 bpm, respiratory rate is 10/min, blood pressure is 113/63 mmHg, and oxygen saturation on room air is 98%. Physical examination shows charring of the face, singed eyebrows, and coarse breath sounds on inspiration. Which of the following is the most appropriate first step in management?

A) Application of a face mask at 40% oxygen
B) Bilevel positive airway pressure
C) Hyperbaric oxygen therapy
D) Intubation with 100% oxygen delivery
E) Oxygenation with 6 L nasal cannula

A

The correct response is Option D.

The patient is at risk for carbon monoxide (CO) toxicity and impending airway collapse. This patient was involved in a house fire and he has signs of lung injury. He should be intubated for airway protection because of his high potential for respiratory collapse. The patient’s normal-appearing oxygen saturation is a result of CO poisoning leading to carboxyhemoglobin being mistaken for oxyhemoglobin. Getting a carboxyhemoglobin level will assist in the diagnosis. The treatment for CO toxicity is 100% inhaled oxygen, which will lead to dissociation of the CO molecule from hemoglobin.

Hyperbaric oxygen treatment has been proposed as a potential treatment for CO toxicity, but the data are controversial, and treatment may be inappropriate in a burn patient undergoing resuscitation.

Even though the patient has an oxygen saturation of 98%, it is falsely elevated in CO poisoning.

This patient has stridor and is at risk for airway edema and should be immediately intubated.

Bilevel positive airway pressure does not protect the airway.

How well did you know this?
1
Not at all
2
3
4
5
Perfectly
21
Q

A 27-year-old man is evaluated in the intensive care unit for polytrauma 12 hours after sustaining burns on 10% of the total body surface area over the bilateral lower extremities during an industrial accident. The patient is sedated, intubated, and is being resuscitated appropriately. Examination shows the wounds are waxy and thick, but not circumferential. He has a nondisplaced fracture of the left tibia; physical examination of the left lower extremity shows that the toes are cool and cyanotic, which is markedly different from the right foot. Left lower extremity pulses are intact, but the left calf is much larger and firmer. Which of the following is the most appropriate next step in management of the left lower extremity?

A) Ankle brachial index
B) CT angiography
C) Elevation
D) Escharatomy
E) Fasciotomy

A

The correct response is Option E.

Compartment syndrome consists of increased pressure within enclosed compartments, resulting in decreased blood flow. For extremities, the concern is for muscle ischemia, as decreased perfusion is unable to supply the metabolic demand. The five P’s of compartment syndrome (pain, pallor, paresthesia, pulselessness, and paralysis) are a good guideline, but are not entirely helpful in this case. As the patient is intubated, pain, paresthesia, and paralysis are unable to be assessed. Skin pallor is confused by the burn eschar, and pulselessness is an extremely late finding. Compartment syndrome needs to be diagnosed early for intervention to be effective. High clinical suspicion is mandatory for this patient because of his clinical condition and trauma. Although measurement of intracompartmental pressures can be done, his ongoing fluid resuscitation and current clinical examination would prompt an emergent surgical intervention.

Elevation is not adequate treatment for compartment syndrome.

Escharotomy would be inadequate as all four fascial compartments will need to be released. Although a circumferential eschar can cause compartment syndrome, this patient does not have circumferential eschar and has a tibial fracture, which points to requiring fascial release.

CT angiography and ankle brachial index are good evaluations of flow and perfusion; however, they are not appropriate in the time sequence of this patient.

How well did you know this?
1
Not at all
2
3
4
5
Perfectly
22
Q

A 19-year-old man is undergoing open reduction and internal fixation of multiple facial fractures when the nasal endotracheal tube is accidentally disconnected from the breathing circuit. The surgeon, who is performing electrocauterization, witnesses an unexpected flash in the operative field. The surgeon immediately stops the procedure and alerts the anesthesiologist. Smoke and a burning odor coming out of the patient’s airway are noted. Which of the following is the most appropriate next step in management?

A) Decrease the flow of airway gases by half
B) Immediately remove the endotracheal tube
C) Pack the oral and nasal cavities with sponges
D) Perform emergency bronchoscopy
E) Reconnect the endotracheal tube

A

The correct response is Option B.

This surgical team is confronted with a fire in the operating room, involving the airway. The most appropriate next step is immediate removal of the endotracheal tube, without waiting.

The incidence of operating room fires in the United States is estimated to be around 600 cases per year. Fire requires the presence of three components: fuel, an oxidizer, and an ignition source. Common fuels in the operating room include alcohol-containing prepping agents, drapes and bandages, gowns and other personal protection equipment, petroleum jelly, etc. Ignition sources include the electrocauterization lasers, fiberoptic light sources, and defibrillators. The two most common oxidizing agents in the operating room are oxygen and nitrous oxide.

Early warning signs of fire include unexpected flash, flame, smoke or heat, unusual sounds (e.g., a “pop,” “snap,” or “foomp”) or odors, unexpected movement of drapes, discoloration of drapes or breathing circuit, and unexpected patient movement or complaint. The surgical procedure should be immediately halted so the team can evaluate whether fire is indeed present.

In cases of airway fire, the ASA practice advisory recommends immediate (without waiting) removal of the tracheal tube, interruption of flow of all airway gases, removal of all sponges or any other flammable materials from airway, and pouring of saline into the airway. Once the fire is extinguished, subsequent steps consist of re-establishing ventilation, avoiding oxidizer-enriched atmosphere if clinically appropriate, examining of the endotracheal tube for possible fragments left behind in the patient’s airway, and considering bronchoscopy.

How well did you know this?
1
Not at all
2
3
4
5
Perfectly
23
Q

A 25-year-old man sustains a high-voltage electrical injury of the right upper extremity with an entrance wound over the volar aspect of the wrist. If increased compartment pressures are not adequately relieved in time, which of the following muscles is most likely to develop an ischemic contracture?

A) Brachioradialis

B) Extensor carpi radialis brevis

C) Flexor carpi radialis

D) Flexor digitorum profundus

E) Palmaris longus

A

The correct response is Option D.

The above patient did not undergo forearm compartment release and has flexion contractures of the interphalangeal joints of the fingers and thumb due to ischemic necrosis of the flexor digitorum profundus (FDP) and flexor pollicis longus (FPL).

Volkmann ischemic contracture is the end result of untreated compartment syndrome. The deeper muscles in the forearm sustain higher pressure sooner and for longer, causing them to be the most affected by compartment syndrome. The FDP to the middle and ring fingers lies against the ulnar and interosseous membrane in the deep compartment of the forearm. These two muscle segments, in addition to the remaining segments of the FDP to the index and small fingers and the FPL, are the most likely to develop contracture. All other muscles listed are more superficial. While brachioradialis, palmaris longus, extensor carpi radialis brevis, and flexor carpi radialis can all become necrotic in untreated compartment syndrome, FDP and FPL are the most likely to do so due to their depth in the forearm.

How well did you know this?
1
Not at all
2
3
4
5
Perfectly
24
Q

In a patient with burns covering more than 30% of the total body surface area, which of the following parameters is most likely to increase in the first few hours of the pre-resuscitation phase?

A) Cardiac output
B) Peripheral blood flow
C) Plasma volume
D) Systemic vascular resistance
E) Urine output

A

The correct response is Option D.

Extensive burn injuries are characterized by the hemodynamic changes seen in hypovolemia, including decreased cardiac output, decreased peripheral blood flow, and decreased urine output. Decreased plasma volume is seen secondary to extravasation of plasma into the burn wound and surrounding tissues. Increased systemic vascular resistance is found in hypovolemia.

How well did you know this?
1
Not at all
2
3
4
5
Perfectly
25
Q

A 30-year-old man who sustained burns on 35% of the total body surface area 24 hours ago is being treated in the burn unit. The patient was resuscitated according to the Parkland formula and is maintaining adequate urine output. Gastric feeding access was established on initial presentation, and the patient is being fed according to his initial body weight. Which of the following clinical indices is most suggestive of the need to decrease his feeding to trophic feeds?

A) Early operative excision with planned start time in 8 hours
B) Hypotension requiring vasopressin support
C) Mild abdominal distension that is soft to palpation
D) Nasogastric output of 100 mL for the past 24 hours
E) Need for escharotomy

A

The correct response is Option B.

There is no doubt that nutritional support for burn patients is integral. A multidisciplinary approach to nutritional assessment and support is ideal, and general knowledge of the indications of when to delay or decrease enteral feeding is essential.

Since the patient has gastric and not post pyloric feeding access, nil per os (NPO) for eight hours allowing adequate gastric emptying to decrease risk of aspiration is necessary.

Gut mucosal integrity is important during the stress of large trauma including burn, and enteral feeding is the preferred modality of access. However, during times of extreme stress, decreased splanchnic flow can cause poor intestinal perfusion, and there is the possibility of gut ischemia due to increased metabolic demand of the gut. Decreasing the gastric feeds to trophic feeds is recommended to decrease the risk of inducing gut ischemia. Clinical indications for threatened intestinal perfusion include firm, obvious abdominal distension and gastric output greater than 200 mL per day; the patient doesn’t exhibit any of these clinical signs.

Hypotension requiring vasopressor support indicated decreased perfusion, which can lead to possible mismatch of gut perfusion with required metabolic demand. Trophic feeds are recommended for patients who exhibit signs of significant decreased perfusion requiring vasopressor support.

Escharotomy will increase fluid losses, but will not change caloric needs.

How well did you know this?
1
Not at all
2
3
4
5
Perfectly
26
Q

A 22-lb (10-kg), 18-month-old male infant is brought to the emergency department because of second- and third-degree thermal burns on the torso and lower extremities. Examination shows burns on 30% of the total body surface area. Administration of which of the following solutions is most appropriate for initial resuscitation of this patient?

A) Hypertonic saline
B) Normal saline
C) Normal saline with 5% dextrose
D) Ringer’s lactate
E) Ringer’s lactate with 5% dextrose

A

The correct response is Option E.

Fluid resuscitation is critical to combat the inflammatory response that occurs after a large burn and prevent the patient from going into shock. Burn injury leads to a combination of hypovolemic and distributive shock resulting from generalized microvascular injury and interstitial third spacing. The goal of fluid resuscitation is to ensure end-organ perfusion while avoiding intracompartmental edema. Ringer’s lactate solution is a relatively isotonic solution that has been advocated as the key component for almost all resuscitation strategies during the first 24 to 48 hours. It is preferable to isotonic normal saline in large-volume resuscitation because its lower sodium concentration (130 mEq/L vs 154 mEq/L) and higher pH concentration (6.5 vs 5.0) are closer to physiologic levels. Ringer’s lactate also has the potential added benefit of the buffering effect of metabolized lactate on the associated metabolic acidosis.

Toddlers in particular are susceptible to inadequate fluid resuscitation and should be monitored closely to ensure adequate urine output of at least 1 mg/kg/hr. In children with burns greater than 15% total body surface area, weighing less than 20 kg and younger than 2 years of age, 5% dextrose should be added to the resuscitation fluid to prevent life-threatening hypoglycemia. These younger patients have lower hepatic glycogen reserves that can be quickly depleted after such significant injuries.

Hypertonic saline and colloid solutions have been advocated and successfully used by some in large-volume resuscitation, but their use still remains controversial and not universally accepted in initial resuscitation.

How well did you know this?
1
Not at all
2
3
4
5
Perfectly
27
Q

A morbidly obese woman is brought to the emergency department after being found unconscious at home. Her family reports that both of the patient’s arms were pinned beneath her body when emergency medical service responders arrived. Physical examination shows tense swelling of the upper extremities. The now conscious patient reports severe pain to passive extension of the digits. In addition to appropriate urgent operative management, the surgeon should anticipate which of the following?

A) Decreased glomerular filtration rate
B) Hypercalcemia
C) Hyperglycemia
D) Hypokalemia
E) ST-segment elevation

A

The correct response is Option A.

This patient has bilateral upper extremity compartment syndrome. The emergency department examination of concern for a long-standing period of upper extremity ischemia. The surgeon should anticipate rhabdomyolysis and its consequent renal and metabolic disorders. Supportive treatment should be undertaken. Hyperkalemia is common in rhabdomyolysis manifesting as peaked T waves on ECG as serum potassium concentrations rise. Glucose D50W, 50 mL intravenously, plus regular insulin (5 to 10 units) move potassium from the extracellular fluid to the intracellular fluid. Intravenous mannitol increases renal blood flow (GFR), attracts fluid from the interstitium (thereby counterbalancing hypovolemia), increases urinary flow (prevents myoglobin cast obstruction), and functions as a free radical scavenger. Allopurinol may be helpful in reducing the production of uric acid. It also acts as a free radical scavenger.

Hypocalcemia is commonplace in the initial phase of rhabdomyolysis; however, it does not usually require correction because this would increase the risk of intramuscular calcium deposition. Indication for correcting hypocalcemia would be impending seizures.

How well did you know this?
1
Not at all
2
3
4
5
Perfectly
28
Q

A 154-lb (70-kg) man is evaluated 1 hour after sustaining deep partial-thickness burns to 50% of his total body surface area when attempting to light a gas grill. Paramedics administered 2 L of intravenous fluid during ground transport. Using the Parkland formula, which of the following is the starting rate for fluid resuscitation in this patient?

A) 375 mL/hr
B) 437 mL/hr
C) 583 mL/hr
D) 714 mL/hr
E) 875 mL/hr

A

The correct response is Option D.

The Parkland formula estimates the amount of crystalloid fluid needed for resuscitation of the burn patient, over the first 24 hours after injury. Volume needed = 4 mL × mass (kg) × % TBSA × 100. This patient would require 4 × 70 × 50% × 100 = 14,000 mL over the first 24 hours. Half of this, or 7 L, is given in the first 8 hours after injury. Because the patient received 2 L prior to arrival, during the first hour he would still need 5 L over the next 7 hours, or 714 mL/hr. Eight hours after injury, he would receive the other half, over 16 hours, or 437 mL/hr. It should be stressed that the Parkland formula serves as an initial guide to fluid resuscitation, and that actual volumes are titrated up or down, depending on urine output.

How well did you know this?
1
Not at all
2
3
4
5
Perfectly
29
Q

A 42-year-old lineman is evaluated after sustaining a high-voltage electrical injury while working on a transformer. Physical examination shows entrance wounds on the volar aspect of the right distal forearm. He reports numbness and tingling of the little finger and weakness of the grip of the right hand. Which of the following is the most appropriate next step in management?

A) Elevation of the forearm and observation
B) Escharotomy and carpal tunnel release
C) Escharotomy and excision of the burned tissue
D) Fasciotomy of the forearm
E) Fasciotomy and release of Guyon canal

A

The correct response is Option E.

This lineman has developed compartment syndrome involving at least the ulnar nerve and has signs of motor and sensory impairment. Escharotomy is not an adequate release in electrical injuries, which often involve deeper structures, such as the pronator quadratus. Therefore, fasciotomy is required. While a carpal tunnel release should be performed in this case, release of Guyon’s canal and decompression of the ulnar nerve in the forearm, wrist, and hand must be performed.

How well did you know this?
1
Not at all
2
3
4
5
Perfectly
30
Q

A 4-year-old boy is brought to a small community hospital by his grandmother, who reports that he fell into a bathtub filled with hot water while under the care of his stepfather. Physical examination shows deep partial-thickness burns on the lower extremities, including the feet, with a clear line of demarcation on the upper ankles; there are no splash marks. Multiple bruises scattered across the chest and periorbital ecchymoses are noted. Child abuse is suspected. Which of the following is the most appropriate next step?

A) Admit the patient to the community hospital for a full skeletal survey
B) Admit the patient to the community hospital for local wound care
C) Notify local police to apprehend the stepfather
D) Transfer the patient to a burn center by emergency medical services
E) Transfer the patient to a burn center by private vehicle

A

The correct response is Option D.

This child meets several criteria for transfer to a burn center: suspected child abuse, significant burns to the hands or feet, and a history that does not match the physical examination. While local police and the department of social services should be notified about this case, the role of the burn care provider is to make sure that the patient receives appropriate treatment and is transferred or discharged to a safe environment. The child will require assessment of the household by a social worker, as well as a pediatric consult to help look for other occult injuries, such as acute or healing long-bone fractures. Transfer to a burn center by private vehicle, despite the wishes of the family, would be placing the child at excessive risk for further injury. Transfer must be coordinated and performed by emergency medical services.

How well did you know this?
1
Not at all
2
3
4
5
Perfectly
31
Q

A 58-year-old man is brought to the emergency department for frostbite injury of the right thumb. A photograph is shown. Which of the following is the inciting mechanism of the injury?

A) Failure of DNA repair mechanisms
B) Formation of extracellular ice crystals
C) Osteonecrosis secondary to vasoconstriction
D) Polymerization of intracellular amino acids
E) Vascular endothelial damage

A

The correct response is Option B.

Frostbite occurs by the formation of ice crystals in the intracellular and extracellular spaces. During the cooling process, the extracellular ice crystals form, and osmotic pressure increases, dragging water out of the cells. This leads to intracellular dehydration and an increase in intracellular electrolytes, proteins, and enzymes that lead to cell death. Additionally, there is vascular endothelial damage leading to intravascular thrombosis and decreased blood flow. Arteriovenous shunting occurs at the capillary level, and end-organ tissue damage is compounded.

During the warming process, there is an influx of fluid back into the cells, causing intracellular swelling. The warming process also allows reflow, vasodilation, and reactive hyperemia to occur, leading to increased inflammatory mediators causing further cell death.

Frostbitten extremities should be rapidly rewarmed in water at a temperature of 104.0°F (40.0°C). Typically, rewarming can be completed in 20 to 30 minutes. Adjunctive use of anti-inflammatory medications and anticoagulants also has been described. Patience is required in determining which areas need debridement.

How well did you know this?
1
Not at all
2
3
4
5
Perfectly
32
Q

A 154-lb (70-kg), 45-year-old man undergoes excision and skin grafting after sustaining total body surface area burns to over 40% of his body. Using the Curreri formula, which of the following is this patient’s total daily caloric need?

A) 2250 kCal/day
B) 3350 kCal/day
C) 4450 kCal/day
D) 5550 kCal/day
E) 6650 kCal/day

A

The correct response is Option B.

The Curreri formula is used to calculate caloric needs: 25 kCal/kg/day + 40 kCal/%TBSA/day. So 25 × 70 + 40 × 40 = 3350 kCal/day.

How well did you know this?
1
Not at all
2
3
4
5
Perfectly
33
Q

An otherwise healthy 35-year-old woman is brought to the burn unit because of severe scalding injury to 50% of the total body surface area. Physical examination shows circumferential full-thickness burns to the right upper extremity, and absent distal pulses. Capillary refill time in the fingers is more than 3 seconds. Appropriate airway control and fluid resuscitation are initiated. Emergent right upper extremity escharotomies are performed. Despite surgical intervention, the blood flow to the distal extremity remains poor based on capillary refill time and Doppler flows. Which of the following is the most appropriate next step in management?

A) Additional escharotomies
B) Administration of an intravenous fluid bolus
C) Angiography
D) Elevation of the extremity and reexamination in 45 minutes
E) Fasciotomy of the upper extremity

A

The correct response is Option E.

The most appropriate next step in management is to proceed with urgent fasciotomies of the upper extremity. Indications for emergency extremity escharotomy are the presence of a circumferential eschar with impending or established vascular compromise of the extremities or digits. Progressive flow reduction by Doppler ultrasound is the primary indication for escharotomy. When evaluating upper extremities, it is advised to Doppler ultrasound the palmar arch, not the wrist, so as to evaluate distal blood flow. Other indications include decreased capillary refill time (more than 2 seconds), cyanosis, and/or relentless deep pain progressing to numbness (in awake patients). Neurovascular integrity should be monitored frequently and in a scheduled manner. Capillary refill time, Doppler signals, pulse oximetry, and sensation distal to the burned area should be checked hourly. After the escharotomy, any continued increase in capillary refill time, decrease in Doppler signal, or change in sensation should lead to immediate further decompression via fasciotomy. A carpal tunnel release is vital during the fasciotomy.

Escharotomies typically are performed at bedside under sterile conditions with intravenous sedation using electrocautery. The aim is to make surgical incisions through burned eschar to allow expansion of underlying tissues. In extremity escharotomies, full-thickness incisions along medial and lateral mid-axial lines should be made. Escharotomies should be carried to just beyond the area of the full-thickness burn. Digital escharotomies are performed along the mid-axial line between neurovascular bundle and extensor apparatus. The ideal side to perform escharotomy allows for preservation of pinch: thumb requires radial incision only, and the index finger, long finger, ring finger, and little finger require ulnar incisions only.

In the scenario described, further fluid boluses, escharotomies beyond the standard releases, and management would not be appropriate next steps, and would increase the risk of local complications like further ischemia, tissue necrosis/gangrene, or systemic complications like hyperkalemia, metabolic acidosis, and renal failure.

How well did you know this?
1
Not at all
2
3
4
5
Perfectly
34
Q

A 27-year-old man is brought to the emergency department after sustaining second- and third-degree burns to most of the anterior torso and the upper extremities. Which of the following describes the appropriate amount of Ringer’s lactate, according to the Parkland formula, for fluid management of this patient’s condition?

A) 1 mL/% TBSA/kg given over the first 6 hours
B) 2 mL/% TBSA/kg given over the first 8 hours
C) 2 mL/% TBSA/kg, half given over the first 8 hours
D) 4 mL/% TBSA/kg given over the first 6 hours
E) 4 mL/% TBSA/kg given over the first 8 hours

A

The correct response is Option B.

The amount of Ringer’s lactate (mL) needed for initial resuscitation is formulated by the Parkland formula of 4 mL/kg/% the total body surface area (TBSA) burned, with half this volume infused over the first 8 hours from the time of injury. In other words, for the first 8 hours, the patient should receive 2 mL/% TBSA/kg. All other options are not accurate based on the Parkland formula and this patient’s TBSA.

One easy way to determine the hourly rate for the first 8 hours is to multiply the % TBSA and kg, and divide by 4. Because the Parkland formula is 4 mL/% TBSA/kg, with half given over the first 8 hours, the 4 in the Parkland formula can be divided by 16 (2 × 8) to give a denominator of 4.

It is absolutely critical to perform accurate estimation of the burn size and weight, because overzealous fluid administration can lead to fluid creep, compartment syndromes, and pulmonary complications.

How well did you know this?
1
Not at all
2
3
4
5
Perfectly
35
Q

An otherwise healthy 22-year-old man is brought to the emergency department after sustaining full-thickness skin loss to the hands. Two-stage reconstruction is planned using a bilaminate neodermis (Integra), followed by skin grafting and simple gauze dressings. Which of the following is the average time for maturation (time from application to removal of silicone layer) of Integra with simple gauze dressings only?

A) 1 week
B) 3 weeks
C) 6 weeks
D) 9 weeks
E) 12 weeks

A

The correct response is Option B.

Integra is a bilaminar skin substitute, composed of a silicone outer layer (mimicking the epidermis) and a biologic scaffold for the inner, dermal layer. Typical maturation (time from application to removal of silicone layer) is 21 days. This process can be accelerated with the use of negative-pressure dressings, but that is not described in this scenario.

How well did you know this?
1
Not at all
2
3
4
5
Perfectly
36
Q

A 26-year-old lineman is brought to emergency department after accidentally grabbing a high-voltage power line. The most severe injury is seen at which of the following anatomical locations?

A) Chest wall
B) Neck
C) Shoulder
D) Upper arm
E) Wrist

A

The correct response is Option E.

Severity of injury is proportional to the cross-sectional area of tissue able to carry current. Thus, the most severe injuries are seen at the wrists and ankles, with decreasing severity proximally. The extremities are the most frequently injured body parts, with the upper extremity predominating.

Most high-voltage injuries occur in workers on the job, so the voltage is known. Temperature increase parallels changes in amperage with tissue temperature being a critical factor in the magnitude of tissue damage. Tissue resistance from lowest to highest is nerve, blood vessels, muscle, skin, tendon, fat, and bone. Deep tissue seems to retain heat so that periosseous tissues, especially between two bones, often sustain a more severe injury than more superficial tissue. The associated macro- and microscopic vascular injury seems to occur nearly immediately and is not reversible. Alternating current (AC) causes tetanic muscle contraction, which may either throw victims away from contact or draw them into continued contact with the electrical source, the latter being more common, given our propensity to grasp at objects and the greater strength in our forearm flexors relative to extensors. This effect is often described as the “no-let-go” phenomenon. Altered levels of consciousness are reported in about half of high-voltage injuries. The tissue injury in electrical burns seems to be a combination of thermal and nonthermal mechanisms. If enough heat is generated, the tissue heats to supraphysiologic temperatures causing denaturation of macromolecules, which is usually irreversible. Electroporation refers to the formation of aqueous pores in lipid bilayers exposed to a supraphysiologic electric field. The applied electric field alters the transmembrane potential, with muscle fibers and nerves being the most susceptible. Subsequent pore formation likely allows calcium influx into the cytoplasm, thereby triggering apoptosis and cell death. Electroporation can therefore induce cell necrosis in the absence of heating.

Transmembrane protein molecules contain polar amino acid residues that can change orientation in an electric field. This effect, known as electroconformational protein degradation, may be irreversible and form yet another mechanism of nonthermal injury.

How well did you know this?
1
Not at all
2
3
4
5
Perfectly
37
Q

A 72-year-old man with advanced congestive heart failure who recently received a left ventricular assist device (LVAD) comes to the office with an ulcerated mass in the mid-parietal region. Punch biopsies reveal squamous cell carcinoma of the skin. The patient has a history of bilateral temporal and midline craniotomies for resection of symptomatic meningiomas. Wide local excision of the tumor creates a scalp defect measuring 8 cm in diameter, with calvarial bone denuded of periosteum at its base. Which of the following is most appropriate for coverage of this patient’s defect?

A) Dermal regeneration template, followed by skin autograft
B) Fasciocutaneous free tissue transfer
C) Full-thickness skin autograft
D) Interpolated scalp flaps, with skin autograft to cover the secondary defect
E) Pericranial flap, covered with skin autograft

A

The correct response is Option A.

The most appropriate coverage of the oncologic defect in this patient is with a dermal regeneration template, such as Integra, followed by skin autograft.

Integra is a synthetic bilaminate neodermis composed of a collagen lattice covered with a thin silastic sheet. A single-layer version (collagen only) is also available, allowing stacking of the product for increased soft-tissue thickness. Vascularization of the collagen layer usually occurs in 3 to 4 weeks, at which point the silastic sheet is removed and a thin split-thickness skin autograft is applied. In the absence of pericranium, burring of the exposed calvarium down to healthy bleeding bone is recommended.

A skin autograft applied directly to calvarial bone denuded of periosteal coverage is unlikely to “take.” Interpolated scalp flaps, most likely requiring grafting of a secondary defect (donor site), would be appropriately indicated for coverage of a midparietal 8-cm defect. These are large flaps, based on the major blood vessels supplying the scalp, with an area of undermining that frequently involves the entire scalp. Unfortunately for this patient with multiple previous craniotomies, the resulting scars impose an unacceptably high risk for flap ischemia.

Similarly, a scar-free, well-vascularized pericranial flap large enough to cover the described defect is unlikely to be found in this patient. The safety and success of free tissue transfer in patients depending on LVADs for hemodynamic stability is still to be determined.

How well did you know this?
1
Not at all
2
3
4
5
Perfectly
38
Q

An otherwise healthy 35-year-old man is exposed to subzero temperatures for 24 hours. After initial management of hypothermia and rapid rewarming of the hands, bilateral upper extremity frostbite is evaluated. Physical examination shows severe frostbite of the hands and up to the wrists bilaterally. Which of the following is the most appropriate next step in management?

A) Corticosteroid therapy
B) Heparin therapy
C) Surgical debridement
D) Systemic antibiotic therapy
E) Thrombolytic therapy

A

The correct response is Option E.

The most appropriate next step in management is to consider intra-arterial thrombolytic therapy. The treatment of frostbite has remained essentially unchanged for the past 25 years. Classic management of frostbite injury includes resuscitation, rewarming, and watchful waiting. The outcome is either tissue recovery or progressive gangrene leading to eventual amputation. A variety of maneuvers aimed at advancing the care of patients with frostbite have been attempted, including hyperbaric oxygen, surgical and medical sympathectomy, pharmaceutical agents, and anticoagulation. None of these have resulted in alterations in the management of this disorder. Recent reports have described the use of thrombolytic therapy using urokinase or tissue plasminogen activator (tPA) as a potential therapy for frostbite.

The rationale for this therapy is based on the understanding that tissue injury in frostbite occurs from two distinct components. Initially, tissue freezing and crystal formation occur and then are improved with tissue rewarming. The more significant cause of tissue injury occurs after thawing, and it is the robust local tissue inflammation and coagulation that stimulate microvascular thrombosis and progressive cell death. By reversing local microvascular thrombosis, tPA has been postulated to restore perfusion before irreversible ischemia and necrosis.

Systemic corticosteroids or antibiotics are not indicated at this time. Systemic heparinization is usually employed as an adjunct after intra-arterial thrombolytics have been initiated. However, it is delivered intravenously. Surgical debridement is done in a delayed fashion after the area of frostbite has demarcated, sometimes weeks to months later.

How well did you know this?
1
Not at all
2
3
4
5
Perfectly
39
Q

A 35-year-old man is admitted to the burn unit after sustaining superficial partial-thickness burns involving 25% of the total body surface area. Medical history includes an allergy to sulfonamide. The burns are cleaned, and silver nitrate–soaked dressings are applied. Which of the following is most likely in this patient?

A) Hyponatremia
B) Metabolic acidosis
C) Neutropenia
D) Painful application
E) Thrombocytopenia

A

The correct response is Option A.

Because of the skin’s important function as a microbial barrier, prevention of infection after burn injury is still one of the most difficult challenges in caring for burn patients. The development of effective topical antimicrobial agents has markedly reduced the incidence of invasive burn wound infection and sepsis. Topical therapy should be started after the initial wound debridement. The three most common topical antimicrobial agents are silver sulfadiazine (Silvadene), silver nitrate, and mafenide acetate (Sulfamylon).

Silver nitrate is typically delivered as a 0.5% solution as a wet dressing. Silver nitrate has excellent antibacterial properties and is effective for most Staphylococcus species and most gram-negative aerobes, including Pseudomonas. This agent is typically used when there is a history of sulfonamide allergy or when sensitivity to the other agents has developed. A common use of silver nitrate is in the setting of toxic epidermal necrolysis. Application is painless, but tissue penetration is poor. Concentrations above 5% are cytotoxic to healthy tissues. Because leaching of sodium, potassium, and calcium is common, this effect should be anticipated and replaced appropriately.

Painful application is associated with mafenide acetate (Sulfamylon). Mafenide acetate is delivered as suspension in a water-soluble base. As a result of its solubility, it has excellent tissue penetration and is often used in heavily contaminated wounds with thick eschar. Because of excellent cartilage penetration, it is also the agent of choice with ear burns. Mafenide acetate is highly effective against gram-negative organisms. Adverse effects include hypersensitivity reactions (7% of patients) and inhibition of carbonic anhydrase with a resultant hyperchloremic metabolic acidosis.

Silver sulfadiazine is the most common topical antimicrobial agent used. It has intermediate tissue penetration secondary to its limited water solubility. This agent has a good antibacterial spectrum, a low incidence of development of resistant organisms, and is applied painlessly. Transient leukopenia is a common adverse effect of silver sulfadiazine. This condition is self-limited and does not appear to increase mortality in burn patients. Switching to a different topical agent for a few days will allow the white blood cell count to return to normal.

Thrombocytopenia is not associated with silver nitrate.

How well did you know this?
1
Not at all
2
3
4
5
Perfectly
40
Q

An 18-year-old woman who sustained a flame burn involving 50% of the total body surface area is resuscitated to a stable cardiovascular and respiratory status. Four days after injury, she undergoes tangential excision and xenografting of all burned areas. Following surgery, the patient returns to the ICU intubated and ventilated. She has thick pulmonary secretions. She received 2 units of packed red blood cells during surgery. Vital signs are as follows:

Temperature99.5°F (37.5°C)

Heart Rate130 bpm

Respiratory Rate22/min

Blood Pressure80/50 mmHg

Oxygen saturation is 96% on 40% FIo2. Cardiac output is 6 L/min, and urine output is 0.1 mL/kg/h. Which of the following is the most likely explanation for these abnormal findings?

A) Acute respiratory distress syndrome
B) Hypovolemic shock
C) Pneumonia
D) Pulmonary embolism
E) Sepsis

A

The correct response is Option B.

The most likely explanation for this patient’s abnormal physiology is hypovolemic shock. The patient just underwent tangential excision of a 50% total body surface area burn, and marked blood loss is to be expected. She received 2 units of packed red blood cells, but this is unlikely to be adequate for such a large burn excision. In addition, her vital signs are typical for hypovolemic shock. Sepsis and acute respiratory distress syndrome (ARDS) are often seen in patients with large burns, but they are usually seen later in the hospital course. Sepsis is associated with fever and a high cardiac output. ARDS is associated with previous large-volume transfusions and lung injury, and should not cause hypotension in isolation. It is also associated with more severe hypoxia. Pneumonia and pulmonary embolism are also associated with a more profound hypoxia than this patient exhibits and are usually seen later in a burn patient’s hospital course.

How well did you know this?
1
Not at all
2
3
4
5
Perfectly
41
Q

A 27-year-old man is brought to the regional burn center 4 hours after he got lost in a snowstorm while he was hiking. Physical examination shows severe frostbite of the feet, purple coloring of the toes, heavy blistering, and marked edema. On Doppler examination, pulses are absent bilaterally. He sustained no other injuries and is otherwise healthy. In addition to warming and pain control, which of the following is the most appropriate next step in the treatment of this patient?

A) Angiography
B) Fasciotomy
C) Hyperbaric oxygen therapy
D) Intravenous administration of heparin
E) Technetium-99 triple-phase scanning

A

The correct response is Option A.

Frostbite is a common injury to the homeless and outdoor adventurers. Direct injury is caused by extracellular freezing of tissues with significant changes of the osmotic gradient of cells which can cause significant electrolyte imbalances. Many inflammatory mediators, including thromboxanes, prostaglandins, histamine, and bradykinin are released which lead to significant edema, endothelial injury, and tissue damage.

The mainstay of frostbite injuries, regardless of severity, includes rewarming, pain control, administration of tetanus prophylaxis, and frequent dressing changes and wound care. In cases of severe frostbite injuries with absent pulses, emergent angiography and infusion of tissue plasminogen activator (tPA) have been shown to significantly decrease the rate of amputation if administered within 24 hours of the onset of frostbite.

Although fasciotomy might be necessary after reperfusion, it is not indicated as a first-line therapy for frostbite. Likewise, heparin and hyperbaric oxygen have not been shown to improve the outcomes in frostbite injuries.

Technetium-99 triple-phase scanning can accurately estimate the level of eventual amputation required if performed in the first several days, but it is not a therapeutic modality and would delay the angiography and administration of tPA if indicated.

How well did you know this?
1
Not at all
2
3
4
5
Perfectly
42
Q

A 165-lb (75-kg), 40-year-old man is brought to the emergency department 3 hours after sustaining first-degree burns to the hands and second- and third-degree burns to the entire anterior thorax and both anterior and posterior lower extremities. According to the Parkland formula, administration of which of the following is the most appropriate method of initial fluid resuscitation in this patient?

A) Hypertonic saline solution 253 mL/hr for 5 hours
B) Hypertonic saline solution 1181 mL/hr for 8 hours
C) Ringer’s lactate 506 mL/hr for 24 hours
D) Ringer’s lactate 1013 mL/hr for 8 hours
E) Ringer’s lactate 1620 mL/hr for 5 hours

A

The correct response is Option E.

Massive injury and burns result in a systemic inflammatory response with resultant leakage of fluid into the interstitial space. Large fluid shifts can decrease perfusion to vital organs and inadequate resuscitation will result in acidosis, oliguria, and relative polycythemia. Fluid replacement is based on the observation that intravascular fluid loss into the interstitium is relatively constant during the first post-injury day. All fluid, therefore, is administered at a constant rate to avoid excessive interstitial edema. Patients are monitored for an adequate clinical response by measurement of hourly urine output (0.5–1 mL/kg/hr for adults) with adjustments as needed. Invasive monitoring may be necessary in elderly patients, patients with cardiac dysfunction, or patients with severe pulmonary injury.

A variety of fluid resuscitation formulas are available to guide the initial management of fluid replacement. The Parkland formula and its variations have become the standard methods for resuscitation. Isotonic crystalloid, Ringer’s lactate in particular (sodium concentration of 130 mEq/L) is the fluid of choice. The Parkland formula directs the resuscitation as follows: 4 mL of fluid × patient weight in kilograms × total body surface area (TBSA) percentage of second- and third-degree burns. Half of this total volume is delivered in the first 8 hours and the second half over the ensuing 16 hours. In a delayed presentation (3 hours in this clinical example), half of the total volume must be delivered within the 8-hour window (remaining 5 hours in this example).

The TBSA is calculated using the “Rule of Nines,” as follows:

Head/Neck = 9%

Each upper extremity = 9%

Anterior thorax = 18%

Posterior thorax = 18%

Each lower extremity = 18%

Groin = 1%

In this patient example, the TBSA of burn is 54%: Anterior thorax (18%) + each lower extremity (18% + 18%). The Parkland formula considers only second- and third-degree burns for calculation of resuscitation volume; therefore, the first-degree burns in this case are ignored.

In this clinical example, 4 mL × 75 kg × 54% TBSA = 16,200 mL total fluid over 24 hours. The patient will require 8100 mL in the first 8 hours and 8100 mL over the ensuing 16 hours. Because the patient presented 3 hours post burn, he will require 8100 mL over the next 5 hours which gives an hourly rate of 1620 mL/hr.

How well did you know this?
1
Not at all
2
3
4
5
Perfectly
43
Q

An otherwise healthy 25-year-old chef comes to the office 2 hours after sustaining scald burns when she accidentally spilled a large pot of soup. She says she washed the area immediately and dressed the burns with silver sulfadiazine. She is alert and her condition is stable. Physical examination shows partial-thickness burns on the lower abdomen, perineum, external genitalia, and anterior thighs involving approximately 15% of the total body surface area. Which of the following is the most appropriate next step in management?

A) Administration of oral antibiotics
B) Outpatient care with silver sulfadiazine
C) Parkland formula fluid resuscitation
D) Referral to the inpatient burn center
E) Split-thickness autografting

A

The correct response is Option D.

The patient described with a partial-thickness burn greater than 10% of the total body surface area (TBSA) and a burn to the perineum meets the criteria for referral to the burn center. Other criteria for burn center referral, as advised by the American Burn Association, include burns that involve the face, hands, feet, genitalia, perineum, or major joints in both young or old patients (younger than age 5 years or older than age 60 years); third-degree burns in any age group; inhalation injury; electrical and lightning burns; chemical burns (especially hydrofluoric acid burns); and any patients with preexisting medical or social conditions that could adversely affect outcomes. Also, children admitted to a hospital without pediatric specialty care should be transferred, if possible.

Studies suggest that outcomes of complex burns improve at high-volume care centers. Most types of small burns can be managed well as outpatient cases, especially by motivated, healthy patients. Others may be eligible for home care to help with dressings if needed. High-volume fluid resuscitations, such as the Parkland formula, are employed for burns greater than 20% TBSA, as the inflammatory response mechanisms that necessitate high-volume resuscitations rarely occur with burns under 20% TBSA. Usually, oral fluids and/or modest intravenous supplementation are sufficient. Autografting should be reserved for deep or function-impeding burns that fail to respond to initial optimal burn wound care.

How well did you know this?
1
Not at all
2
3
4
5
Perfectly
44
Q

A 25-year-old woman is brought to the emergency department after sustaining deep partial-thickness and full-thickness burns to the face, neck, chest, back, and bilateral upper extremities in a grease fire. The patient is intubated and resuscitated, and the wounds are managed surgically. Which of the following is the most appropriate position to splint the burned areas?

A) Elbow extended at 180 degrees
B) Hands in intrinsic minus position
C) Neck flexed at 45 degrees
D) Shoulder abducted at 60 degrees
E) Wrist flexed at 10 degrees

A

The correct response is Option A.

The most appropriate position to splint the different burned areas is neck in slight extension, shoulder fully abducted to about 90 degrees, elbow fully extended at 180 degrees, wrist in neutral or slightly extended, and hands in intrinsic plus position or position of function. This is done to prevent contractures that would pull these joints into positions that would lead to functional deficits.

How well did you know this?
1
Not at all
2
3
4
5
Perfectly
45
Q

A 59-year-old postal worker comes to the emergency department because of numbness and discoloration of the right thumb. He says he was outdoors for 8 hours in a temperature of -15°F (-26°C) and had taken ibuprofen before arrival. Physical examination shows distal bluish skin without capillary refill. A photograph of the thumb is shown. Which of the following is the most appropriate initial management?

A) Debridement or amputation of dead tissue, if necessary, to avoid infection
B) Debridement of any hemorrhagic blisters
C) Discontinue antiprostaglandins
D) Observe and allow the tissue to fully demarcate
E) Rapid rewarming with radiant heat

A

The correct response is Option D.

The decision to allow tissue to fully demarcate is essential in the scenario described because it is difficult to determine which tissue may survive in the immediate postinjury period. This period may be 6 months.

Clear blisters are debrided because they contain high concentrations of inflammatory mediators. Inflammatory mediators lead to greater tissue injury. Hemorrhagic blisters are not debrided because this may cause exposure of deep structures and increase the risk of desiccation and subsequent necrosis.

Early debridement and amputation are necessary if soft-tissue infection develops during the waiting period; otherwise, they are not indicated. Premature amputation risks sacrifice of potentially salvageable tissue.

Antiprostaglandins, either topical (such as 70% aloe cream) or oral (ibuprofen 12 mg/kg), are beneficial to block the cascade of arachidonic acid by cyclooxygenase and the inflammatory process. Beneficial effects of antiprostaglandins include vasodilatation and antiplatelet aggregation.

Rapid rewarming decreases further tissue damage as it halts both direct injury and continued release of secondary mediators. Submersion of the injured part in 104°F (40°C) water for 15 to 30 minutes is ideal. This maneuver stops ice crystal formation and reverses the deleterious effects of vasoconstriction. Use of radiant heat sources in frostbite can lead to iatrogenic injury due to uneven thawing and, in unusual cases, secondary thermal burn to insensate tissue. Before and after photographs are shown.

How well did you know this?
1
Not at all
2
3
4
5
Perfectly
46
Q

A 25-year-old woman with burns on 85% of the total body surface area undergoes staged wound excision but shows limited donor sites for skin grafting. Cultured epidermal autografts (CEAs) are prepared to help resurface the wounds. Which of the following properties is the primary advantage of the use of CEAs over split-thickness skin grafts?

A) Cultivation period of 1 week
B) Expansion of donor keratinocytes
C) Negligible production cost
D) Stable coverage of the lesion
E) Use of autologous materials during cell culture

A

The correct response is Option B.

Theoretically, cultured epidermal autografts (CEAs), also known as cultured keratinocytes, are an attractive option to help resurface large wounds, such as in the massively burned patient with limited donor sites. After obtaining a small skin biopsy, tissue is then processed ex-vivo by a commercial tissue-engineering laboratory. Within 3 weeks, keratinocytes can be expanded 10,000-fold and are ready for grafting. CEAs, however, must be grown with murine fibroblasts and fetal calf serum, both of which contain xenogeneic proteins that survive at the time of transplantation and may account for ?rejection? of these autografts. Furthermore, CEAs lack a dermal component and are extremely fragile, susceptible even to mild sheer forces. Finally, CEAs are very expensive, costing as much as ,000 for every 1% of the total body surface area that is ultimately covered.

How well did you know this?
1
Not at all
2
3
4
5
Perfectly
47
Q

A 25-year-old woman who sustained burns to 85% of the total body surface area in a house fire undergoes staged excision of the wound but has limited donor sites for skin grafting. Cultured epidermal autografts are applied for resurfacing. Which of the following is the most significant advantage of this procedure?
(A) Cost effectiveness
(B) Expansion of donor keratinocytes
(C) Immediate availability of autogenous materials
(D) Short cultivation period
(E) Stable coverage of grafted wounds

A

The correct response is Option B.

Cultured epidermal autografts (CEAs), also known as cultured keratinocytes, are theoretically attractive to help resurface large wounds, such as in the massively burned patient with limited donor sites. After obtaining a small skin biopsy, tissue is then processed ex vivo by a commercial tissue-engineering laboratory. Within three weeks, keratinocytes can be expanded 10,000-fold and are ready for grafting. CEAs, however, must be grown with murine fibroblasts and fetal calf serum, both of which contain xenogeneic proteins, which survive to transplantation and may account for “rejection” of these autografts. Furthermore, CEAs lack a dermal component and are extremely fragile, susceptible even to mild sheer forces. Finally, CEAs are very expensive, costing as much as $13,000 for every 1% total body surface area ultimately covered.

How well did you know this?
1
Not at all
2
3
4
5
Perfectly
48
Q

A 40-year-old man has second- and third-degree burns involving 55% of the total body surface area. Which of the following immunologic responses is most likely in this patient during the first week after injury?

A) Downregulation of integrins
B) Downregulation of cytokines tumor necrosis factor (TNF)-a and interleukins 1 and 8
C) Increased B-lymphocyte function
D) Increased levels of circulating immunoglobulins
E) Increased T-suppressor lymphocyte function

A

The correct response is Option E.

Nearly all aspects of immune function are affected by thermal injury, and the effect is directly related to the extent of the injury. Burns that are on greater than 30% of the total body surface area result in a greater systemic inflammatory response because of circulating cytokines and immune mediators.

Both humoral and cellular-mediated immunity are impaired by thermal injury and are manifested by diminished activation of complement and depressed levels of circulating immunoglobulins; upregulation of integrins and the cytokines TNF-a and interleukins 1 and 8 (IL-1, IL-8); decreased B-lymphocyte, natural killer cell, and T-helper lymphocyte function; and an increased number and activity of T-suppressor lymphocytes. These changes normalize during the ensuing 2 to 3 weeks in patients whose course is uncomplicated.

TNF-a and IL-1 and IL-8 increase neutrophil chemotaxis into the wound as well as the upregulation of cell surface integrin receptors. These migrating neutrophils can degranulate, releasing proteases and oxygen-free radical species, leading to further tissue damage. Experimental studies using monoclonal antibodies directed against cell surface receptors have shown diminished tissue necrosis and a subsequent decrease in the surface area of burn.

Diminished phagocytosis and reduced activation of complement result in diminished antibody-presenting complexes and membrane-attacking complexes. Along with diminished T-helper lymphocyte function, this results in decreased B-lymphocyte numbers, lymphocyte function, and levels of circulating immunoglobulins, especially immunoglobulin G. In addition to decreased production, circulating antibodies are also lost due to increased protein turnover and plasma leakage resulting from increased capillary permeability. Serum immunoglobulin levels gradually return to normal during the ensuing 2 to 4 weeks as the patient recovers.

Impairment of cell-mediated immunity resulting from alterations in T-lymphocyte function is evidenced by a decrease in T-cell function and number, reduced T-helper and natural killer cell activity, and increased T-suppressor function and number. Impairment in T-cell mediated immunity is demonstrated by delayed rejection of allograft skin, suppression of graft-versus-host response, and skin hypersensitivity reactions. Better understanding of altered T-helper lymphocyte activity and the associated cytokine profiles may allow for immune-directed therapies that may decrease morbidity and mortality.

49
Q

A 19-year-old woman comes to the office for a follow-up examination 18 months after skin grafting for burns over 25% of her total body surface area that includes the face, neck, chest, and shoulders. Physical examination shows persistent restriction of neck excursion and lateral range of motion. A photograph is shown. Which of the following is the most appropriate management?

A) Compression therapy
B) Corticosteroid injection
C) Silicone sheeting
D) Skin grafting
E) Z-plasty tissue rearrangement

A

The correct response is Option E.

The patient described has a condition that is common among burn patients with significant burns to the neck area; that is, contracture band formation, even after primary attempts at skin grafting. Contractures are produced not only by absolute skin loss and/or skin contraction along a straight line scar, but also by the differential growth rate between the burn scar and the rest of the adjacent normal skin and tissues, especially in younger patients.

There are various techniques used to release contractures. Management depends on both the location of the contracture as well as the availability of unaffected adjacent skin. Methods of release include tissue expansion, local and distant flaps, ?alphabet plasties? (Z, W, V-Y, etc.), and skin grafting. The most appropriate option for the patient described is a Z-plasty, especially as she has failed a prior attempt at skin grafting. Through soft-tissue rotation, Z-plasty accomplishes three things: (1) lengthens a contracted scar, (2) breaks up a straight line, and (3) shifts soft-tissue contour.

Compression therapy/splinting is useful as an adjunct therapy but not as a primary treatment of this deformity. The patient described does not have a hypertrophic or keloid scar – it is a contracture band. Therefore, the injection of corticosteroids and silicone sheeting are not indicated in the treatment of this deformity.

The advantage of skin grafts, in general, is the transfer of new skin from a previously uninjured area of the body to the area in need with low donor site morbidity, though full-thickness grafts generally have larger donor site morbidity than split grafts. The disadvantage of skin grafts is that they tend to recontracture, necessitating further release through reoperation, as in the scenario described. Prolonged periods of postoperative physiotherapy and splinting are required to maximize aesthetic and functional results.

50
Q

A 50-year-old woman comes to the emergency department because of an itchy skin rash 2 weeks after administration of trimethoprim-sulfamethoxazole for treatment of a urinary tract infection. Temperature is 99.5°F (37.5°C), pulse rate is 110 bpm, respiratory rate is 28/min, and blood pressure is 95/60 mmHg. Oxygen saturation on pulse oximetry is 96%. Physical examination shows an exfoliating skin rash on 35% of the total body surface area. Which of the following is the most appropriate next step in management?

A) Administration of systemic corticosteroids
B) Full-thickness skin biopsy
C) Intravenous administration of immune globulin
D) Topical application of silver sulfadiazine
E) Transfer to a burn center

A

The correct response is Option E.

A patient who develops an exfoliating rash 1 to 3 weeks after starting a specific medication, such as trimethoprim-sulfamethoxazole (Bactrim), allopurinol, or phenytoin (Dilantin), has toxic epidermal necrolysis syndrome (TENS, also known as Stevens-Johnson syndrome) until proven otherwise. Patients with TENS often have several days’ worth of indolent and nonspecific symptoms, such as malaise, fever, and dysphagia. These symptoms progress rapidly to hemodynamic collapse, skin exfoliation, and mucosal sloughing.

Mortality for patients with TENS is 30%. The proposed mechanism is an acute autoimmune response to the basement membrane of epithelial structures, induced by drug exposure. Incidence is one case per million.

Skin biopsy is pathognomonic but should not delay treatment. Administration of intravenous immune globulin and systemic corticosteroids are both controversial and may have negative effects. Application of topical silver sulfadiazine as a wound care cream is contraindicated because of the potential for exacerbation of the immune response from the sulfa moiety of this agent. Although many modalities of treatment have been proposed, only transfer to a burn center has been universally accepted as a priority because of the critical care and wound care necessary to impact survival.

51
Q

A 34-year-old man comes to the emergency department after sustaining electrical burns to the right upper extremity while working on high-voltage power lines. Physical examination shows full-thickness burns on the right volar forearm involving 3% of the total body surface area. Poikilothermia and pallor are noted over the affected area. Pulses are not palpable. Supplemental oxygen is administered, and fluid resuscitation is initiated. Which of the following is the most appropriate next step in management?

A) Elevation of the arm
B) Escharotomy of the volar forearm
C) Fasciotomy of the volar forearm only
D) Fasciotomy of the volar forearm and carpal tunnel release only
E) Fasciotomy of the volar forearm, including decompression of the pronator quadratus, and carpal tunnel release

A

The correct response is Option E.

Following high-voltage injury with full-thickness or partial-thickness skin loss, patients may develop compartment syndrome. In such cases, immediate decompression is mandated. Patients typically show the pentad of pain, paresthesias, pallor, poikilothermia, and pulselessness. In addition to releasing the eschar and the fascia of the forearm, carpal tunnel release is important to decompress both the palmar arch and median nerve. Exploration of the deep compartment of the forearm, including the pronator quadratus, is essential, as tissue injury may increase in proximity to the radius and ulna due to the heat generated by passage of the electrical current. The other options are helpful but not sufficient.

52
Q

A 57-year-old man develops exposure keratitis, corneal ulceration, and bilateral upper and lower eyelid ectropions 2 months after sustaining burns to the head and neck during a gas grill explosion. A photograph is shown. A tracheostomy was performed at the time of the accident for inhalation injury. Which of the following is the most definitive treatment modality for this patient’s condition?

A ) Adhesion tarsorrhaphy

B ) Bilateral tarsoconjunctival flaps

C ) Frost sutures

D ) Lateral tarsal strip canthoplasty

E ) Staged upper and lower ectropion releases with skin grafts

A

The correct response is Option E.

The patient described requires definitive treatment of the burn ectropions, which involves release of the ectropion and resurfacing of the orbicularis muscle with high-quality skin that will resist contraction. Although full-thickness skin grafts will have less of a tendency to contract than

split-thickness skin grafts, the shoulder and supraclavicular fossa are superior to the groin as donor sites in terms of texture and color match. Photographs are shown.

Tarsorrhaphy alone will provide temporary coverage of the cornea but neither addresses the underlying pathophysiology nor provides definitive correction of the ectropion. Simultaneous correction of all four eyelids does not permit adequate release of the ectropions, which must be overcorrected before grafting.

Although Frost sutures, tarsoconjunctival flaps, and lateral tarsal strip canthoplasties will provide temporary corneal protection, these procedures do not address the underlying problem, which is contracture of eyelid, forehead, and cheek skin, secondary to burn injury.

53
Q

A 55-year-old man is brought to the emergency department after sustaining electrical burns. He has numbness of the left hand; pulse in the hand and sensation to touch are diminished. Which of the following is the most effective management to restore perfusion to the left hand?

A ) Anticoagulation

B ) Embolectomy

C ) Burn excision

D ) Escharotomy

E ) Fasciotomy

A

The correct response is Option E.

The patient described has a circumferential electrical injury to the forearm consistent with compartment syndrome. Fasciotomy is indicated.

Acute burn injury exceeding 20% can lead to a significant systemic response, with release of vasoactive mediators, third spacing, increased metabolic requirement, and immune suppression. Surgical debridement and closure of the wound can limit the inflammatory process.

Anticoagulation and embolectomy do not address the underlying problem, which is increased compartment pressure in a confined space. Burn excision is a method of removing devitalized burned tissue, usually in a tangential fashion down to the fascia, followed by split-thickness skin grafting. This is not a treatment for compartment syndrome.

When there is circumferential injury of an extremity or in deep burns to the chest wall, escharotomy, or creating incisions through the burn eschar, is important in releasing constriction and allowing circulation to the extremity or airflow to the chest. Incisions must extend beyond the zone of the burn injury to assure complete release of the constriction to restore circulation. Escharotomy is performed within the first 24 hours of admission.

54
Q

Which of the following skin substitutes contains foreskin-derived neonatal human fibroblasts and keratinocytes?

A ) AlloDerm

B ) Apligraf

C ) Biobrane

D ) Integra

E ) Surgisis

A

The correct response is Option B.

Apligraf is a permanent, biosynthetic, bilayered living construct of cultured foreskin-derived neonatal human keratinocytes and fibroblasts. They are cultured on a matrix consisting of bovine-type collagen.

AlloDerm is a human cryopreserved, acellular, cadaveric, de-epidermalized dermis. The complex is immunologically inert and becomes repopulated with host fibroblasts and endothelial cells.

Biobrane contains Type I porcine collagen peptides in a bilaminate of silicone film and nylon fabric.

Integra is a temporary bilaminate composed of silicone and a matrix of cross-linked bovine tendon collagen and shark-derived glycosaminoglycans.

Surgisis is derived from porcine small intestine and is processed into a biocompatible three-dimensional, extracellular matrix composed of collagen, noncollagenous proteins, and other biomolecules.

55
Q

A 30-year-old woman who is morbidly obese is admitted to the burn unit with partial-thickness burns on 40% of the total body surface area involving the trunk and lower extremity. Maintenance of which of the following is the most appropriate measure to guide proper fluid management of the patient?

A ) Arterial systolic pressure greater than 90 mmHg

B ) Cardiac output greater than 5 L/min

C ) Mean arterial pressure greater than 55 mmHg

D ) Pulse rate less than 120 bpm

E ) Urinary output of 0.5 mL/kg/h

A

The correct response is Option E.

Routine vital signs, such as blood pressure and heart rate, can be very difficult to interpret in patients with large burns. Catecholamine release during the hours after the burn can support cardiac output despite the extensive intravascular depletion that exists. The formation of edema in the extremities can limit the usefulness of noninvasive blood pressure measurements. Evaluation of arterial line pressures is subject to error from peripheral vasospasm from the high-catecholamine state. Tachycardia, normally a clue to hypovolemia, can be secondary to pain and is also almost universally present from the adrenergic state. Moreover, placement of a central line to measure the cardiac output in a morbidly obese patient may pose risks and should be avoided unless the burns involve the upper part of the body, which will lead to edema in the later stages of resuscitation. Blood pressure is not an accurate measure of tissue perfusion.

Hourly urine output is a well-established parameter for guiding fluid management. The rate of fluid administration should be titrated to a urine output of 0.5 mL/kg/h or approximately 30 to 50 mL/h in most adults and older children (> 50 kg [110 lb]). The urge to maintain urine output at rates greater than 30 to 50 mL/h should be avoided. Fluid overload in the critical hours of early burn management leads to unnecessary edema and pulmonary dysfunction. It can necessitate morbid escharotomies and extend the time required for ventilator support. Several complicating factors exist with monitoring urine output as a guide for volume status and end-organ perfusion. The presence of glycosuria can result in an osmotic diuresis and lead to artificially elevated urine output values. Performing a urinalysis at some point during the first 8 hours is prudent, especially for patients with larger burns, to screen for this potentially serious overestimation of the intravascular volume.

56
Q

A 42-year-old man is brought to the emergency department after being rescued from an avalanche. History includes type 2 diabetes mellitus that is well controlled by diet. He has smoked one pack of cigarettes daily for the past 10 years. Physical examination shows erythema, edema, and blistering of the right lower extremity extending from the distal tibia to the tips of the toes. Which of the following is the most appropriate first step in management?

A ) Debridement and negative pressure wound therapy

B ) Hyperbaric oxygen therapy

C ) Intravenous administration of heparin

D ) Primary amputation

E ) Rewarming in a water bath 104 °F (40 °C)

A

The correct response is Option E.

The mainstay of treatment for frostbite injuries is rapid rewarming by submersion of the affected body part in a water bath maintained at a constant temperature of 104 to 108 °F (40 to 42 °C). Frostbite involves the formation of extracellular ice crystals that produce cellular damage. Rapid rewarming is the primary treatment for frostbite and has not changed in nearly three decades. The optimal temperature was demonstrated in controlled experiments by Entin and Baxter in 1952. Rewarming may take 20 to 40 minutes and is deemed adequate when there is evidence of distal perfusion, such as a blush. Addition of an antibacterial soap solution to the water bath is often recommended. The affected body part should then be splinted and elevated. It may take weeks or months for tissues to fully declare themselves. Delayed amputation is often required in severe frostbite injuries.

Debridement and negative pressure wound therapy have not been shown to be an effective initial step in management compared with rapid rewarming, but they may have a role in secondary wound management.

Hyperbaric oxygen (HBO) therapy has been reported as an adjunctive treatment to frostbite, but mostly as case reports. HBO may have a role in limiting the progressive tissue necrosis that follows the acute injury, but the lack of a prospective randomized controlled study has prevented it from achieving wide acceptance.

Heparin and other anticoagulants and thrombolytics have been tried with some variable success, but they should be instituted only after rapid rewarming measures have been applied. There is experimental and clinical evidence to suggest that one of the mechanisms of tissue injury is

thrombosis of the microvasculature that occurs from direct tissue injury and is mediated through free radicals.

Primary amputation would not be appropriate as an initial step in management, as this would commit the patient to a morbid operation with significant disability.

Gradual rewarming should not be used, as this has been associated with greater tissue injury.

57
Q

A 32-year-old man is brought to the emergency department after being lost during a snowstorm in the mountains for 24 hours. Physical examination shows significant edema, loss of sensation, grayish blue discoloration, and hemorrhagic blisters on both hands and feet. Which of the following is the most appropriate initial management?

A ) Debridement of the hemorrhagic blisters

B ) Oral administration of ibuprofen

C ) Rapid cycles of freezing and thawing

D ) Rapid rewarming using a radiant heat source

A

The correct response is Option B.

Ibuprofen provides antiprostaglandin activity to limit the potential for secondary mediator damage.

Debridement of hemorrhagic blisters may cause exposure of the deeper structures and run the risk of desiccation and subsequent necrosis. Traditionally, early and aggressive debridement and amputation are avoided. The development of deep, dry gangrene can be allowed to declare the specific regions that must undergo amputation. On the other hand, triple-phase scanning can be used to delineate viable tissue early on and obviate prolonged demarcation.

Cycles of freezing and thawing lead to greater inflammatory mediator release and, in the long run, greater tissue injury.

Rapid warming decreases further tissue damage by halting both direct injury and continued release of secondary mediators. This occurs by submersion of the injured part in 104 °F (40 °C) water for 15 to 30 minutes. Use of radiant heat sources in frostbite can lead to iatrogenic injury because of uneven thawing and secondary thermal burn to insensate tissue.

58
Q

A 4-year-old boy has wound cellulitis, pneumonia, and bacteremia one week after sustaining burns on 38% of the total body surface area in a house fire. He undergoes debridement of the wounds. Grafting is performed to reconstruct the resulting defects. Debrided tissue is sent for culture. While awaiting the results of culture, which of the following is the appropriate empiric antibiotic therapy?

A ) Ampicillin-sulbactam, ciprofloxacin, and tobramycin

B ) Penicillin G, clindamycin, and gentamicin

C ) Vancomycin and piperacillin-tazobactam

D ) Vancomycin, piperacillin-tazobactam, and amphotericin

E ) Vancomycin, piperacillin-tazobactam, and fluconazole

A

The correct response is Option C.

The most common bacteria causing burn wound infections include methicillin-resistant Staphylococcus aureus (MRSA), Pseudomonas, and Klebsiella. Empiric antibiotic therapy for the patient described would need to cover these gram-positive cocci and gram-negative rods.

Of the antibiotics listed, only vancomycin covers MRSA. Piperacillin-tazobactam is the most appropriate choice for pseudomonal coverage. Antifungal coverage with fluconazole or amphotericin is not necessary and is not supported by the literature. Antifungal prophylaxis remains controversial.

59
Q

A 62-year-old woman comes to the office for consultation regarding problems with her left eye (shown) 6 months after sustaining a 25% total body surface area (TBSA) burn that involved the face and neck. She reports that the upper eyelid turns outward, that she cannot close the eye completely, and that the eye tears excessively. These symptoms have persisted despite skin grafting several months ago by another surgeon. Physical examination confirms ectropion of the upper eyelid, lagophthalmos, and epiphora of the left eye. Which of the following is the most appropriate next step in management?

A ) Insertion of a gold weight

B ) Paramedian forehead flap

C ) Permanent lateral tarsorrhaphy

D ) Reverse tarsoconjunctival flap

E ) Skin grafting

A

The correct response is Option E.

Patients with significant burns to the periocular area commonly have ectropion with corneal exposure, even after primary attempts at skin grafting. It is clear that damaged contracted skin has left this patient €œskin short, € thereby preventing complete closure of the eyelid. The optimal method of eyelid reconstruction in this patient is a release of the burn scar contracture with placement of nonburned, thin, pliable skin to the area in order to bring tissue to the area, thereby correcting the underlying problem. Although multiple options for reconstruction exist, the most reasonable would involve regrafting the upper eyelid. While the literature €œdogma € has been to graft the upper eyelid with split grafts and the lower eyelid with full-thickness grafts, this has not been definitely proven one way or the other by the available data. However, repeat grafting (either with thin or full-thickness grafts) to correct this problem has been used successfully to achieve good outcomes.

A gold weight is used for the paralytic eyelid after facial nerve injury.

A permanent lateral tarsorrhaphy can be performed but will narrow the ocular aperture and can be irritating to the patient.

Forehead flaps have been described to reconstruct eyelids, especially large full-thickness defects. Their biggest downside, however, is the thickness of the tissue transferred, which certainly does not reapproximate the thickness of the native upper eyelid. It also involves a two-stage procedure. The patient described has suffered burns to her face and neck, including the forehead, which renders this method of reconstruction suboptimal.

A reverse tarsoconjunctival flap can be used to reconstruct upper eyelids; however, it is generally used when the lower eyelid is not involved in the injury. It also has the disadvantages of requiring two operations, occluding the visual axis, and can only be performed with a minimum of lower eyelid tarsus, as only 2 to 3 mm is available for transfer. This patient €™s upper eyelid requires more tissue than what this technique can deliver.

60
Q

An unconscious 25-year-old man is brought to the emergency department after being electrocuted while working near high-voltage power lines. The patient is resuscitated. Examination shows a 4 × 3-cm burn on the skin over the left antecubital fossa and significant swelling of the forearm. Which of the following is the most appropriate next step in management?

A ) Dressing of the affected area and observation in the burn unit

B ) Excision and coverage with a local flap

C ) Excision and coverage with a split-thickness skin graft

D ) Fasciotomy of the forearm

E ) Splinting of the hand in the intrinsic plus position

A

The correct response is Option D.

In high-voltage injuries, the electrical current often travels deep into the skin, causing internal damage that may not be readily visible. The current causes tissue damage at the entry point in the skin and along its path through the muscle, nerves, and bone. As electrical current is conducted through the body, heat is generated in direct proportion to the tissue resistance. Because bone has a high resistance, heat is generated rapidly in this area, resulting in deep tissue injury. Compartment syndrome can develop in a patient with an electrical injury because of the deep tissue injury and subsequent subfascial edema. Immediate treatment is aimed at resuscitation of the patient, followed by salvage of the affected limb. The key to the acute management of electrical injuries to the upper extremity is to have a high index of suspicion for potential damage to deeper tissues, even at a distance from the point of contact. The optimal management of electrical injuries to the upper extremity includes initial exploration, decompression (fasciotomy), and aggressive repeated debridement, followed by reconstruction. Fasciotomy serves a dual role as both a therapeutic and diagnostic tool in the treatment of electrical injuries.

61
Q

A 47-year-old man has deep partial-thickness burns of the neck, trunk, and abdominal wall involving 55% of the total body surface area. Which of the following is the most effective immediate method to deliver nutrition to this patient?

A ) Ad lib oral intake with high-calorie protein shakes

B ) Enteral nutrition through gastrostomy tube

C ) Enteral nutrition through nasogastric tube

D ) Parenteral nutrition through central venous catheter

E ) Parenteral nutrition through peripheral intravenous injection

A

The correct response is Option C.

Providing nutrition early in the management of a burn victim is critical to a successful outcome. Since the gastrointestinal tract is typically intact in a burn victim, enteral feeding is the route of choice. In massive burn patients, care must be taken to recognize gastric and/or intestinal ileus, which may complicate and inhibit the advancement of enteral feeds. Nevertheless, burn victims rarely require parenteral nutrition on a long-term basis.

62
Q

A 20-year-old college student is being evaluated because of painful blistering of his toes the morning after he walked two miles in snowy weather, wearing sandals and no socks. Which of the following best describes the primary beneficial effect of ibuprofen for this patient?

A ) Control of edema

B ) Control of pain

C ) Decreased secondary tissue damage

D ) Prevention of antiplatelet aggregation

E ) Speeding of demarcation

A

The correct response is Option C.

Thromboxane A2 is a powerful mediator of the inflammatory process responsible for secondary tissue damage in frostbite injuries. Ibuprofen blocks the cyclooxygenase cascade that results in the production of thromboxane A2. The negative effect of blockade of the cyclooxygenase is decreased production of prostaglandin I2 and E2, which are responsible for vasodilatation and antiplatelet aggregation. Although pain control is a beneficial result of the use of ibuprofen, many patients suffering from frostbite injuries require narcotic pain medications. Demarcation cannot be speeded by the use of ibuprofen and may require up to six months before amputation should be undertaken.

63
Q

A 48-year-old man is brought to the emergency department 40 minutes after he sustained full-thickness burns over 54% of the total body surface area. Weight is 156 lb (70 kg). According to the Parkland formula, which of the following is the most appropriate volume of fluid resuscitation for this patient during the first eight hours?

A ) 945 mL

B ) 1890 mL

C ) 3780 mL

D ) 7560 mL

E ) 15,120 mL

A

The correct response is Option D.

Acute burn injury is characterized by inflammation and impaired microvascular integrity with resulting capillary leak. Therefore, large amounts of fluid are required to maintain tissue perfusion during this period and prevent burn shock. Many formulas have been delineated for acute burn resuscitation to quantify the amount of fluid needed. The Parkland formula, commonly used in the United States, is 4 mL of lactated Ringer solution per kilogram per percentage burn given over the first 24 hours. One half is given in the first eight hours and the second half over the ensuing 16 hours. The total body surface area (TBSA) of burn can be approximated using the rule of nines. The body is divided into regions whose surface areas are multiples of nine: head, 9%; each arm, 9%; torso, 36%; each leg, 18%. In this case, the patient €™s burn is 54% TBSA. The Parkland formula is 4 mL * 70 kg * 54 = 15,120 mL. Half of this (7560 mL) is given over the first eight hours.

64
Q

A 61-year-old obese woman comes to the office for follow-up examination because she has a nonhealing 3-cm-diameter ulcer on the sole of the right foot. Medical history includes hypertension and type 2 diabetes mellitus. A bioengineered skin substitute consisting of a cultured dermal-epidermal matrix (Apligraf) was applied one week ago. The patient has been compliant with a strict non-weight-bearing status of the foot. Current physical examination shows minimal adherence of the graft. Which of the following is the most appropriate next step in management?

A ) Application of hydrogel and gauze dressings

B ) Application of a new biosynthetic graft matrix

C ) Coverage with a sural fasciocutaneous flap

D ) Full-thickness skin grafting

E ) Surgical debridement to fascia

A

The correct response is Option B.

Biosynthetic skin products, particularly Apligraf, have been shown to be effective in treatment of chronic diabetic foot ulcers when used judiciously and appropriately. The largest multicenter, randomized trial of foot ulcer care showed a healing rate of 56% with biosynthetic grafting versus 38% with saline gauze treatment after 12 weeks. Both treatment groups underwent aggressive, sharp debridement and off-loading techniques. The group receiving biosynthetic grafts underwent treatment weekly for a maximum of five grafts. The fibroblasts and keratinocytes in the matrix secrete growth factors and stimulate wound healing and ingrowth of the surrounding tissues. One drawback of biosynthetic skin substitutes is the high cost. If complete wound healing can be achieved, then cost of care is lower over the long term. A single application of a dermal matrix product is not likely to be sufficient in the treatment of a chronic wound. Abandoning the course of treatment at one week would increase the cost of care without the potential to realize the benefit.

Flap coverage would be better suited for covering exposed bone or tendon or for treatment of a wound that has failed more conservative treatments.

Full-thickness grafts are less likely to take than split-thickness grafts in the area of poor wound healing described in this scenario. A split-thickness graft can be used initially instead of biosynthetic grafting and could still be used in the event of failure to heal after a proper treatment course. The advantage of using the biosynthetic instead of autologous grafting is the ability to place the graft in an office setting without anesthesia if the wound bed has been debrided adequately.

Application of moist gauze dressings without completion of a proper treatment course with the graft matrix is likely to result in continuation of this chronic wound.

Debridement of the granulation bed would remove a healthy base for grafting. Debridement would be indicated for presence of necrosis, eschar, or fibrosis €”none of which are present in the adequately debrided wound described.

65
Q

A 5-year-old boy is brought to the emergency department after he sustained burns in a gasoline fire. Physical examination shows burns to 70% of the total body surface area, including the right ear (shown). Which of the following is the most appropriate initial management of the burns to this patient €™s ear?

(A) Application of mafenide acetate (Sulfamylon) cream and auto-amputation

(B) Costochondral reconstruction with a temporoparietal flap

(C) Debridement and coverage with a retroauricular pocket flap

(D) Excision and full-thickness skin grafting

(E) Excision and unmeshed split-thickness skin grafting

A

The correct response is Option A.

Ear burns are usually aggressively managed with mafenide acetate (Sulfamylon) cream, monitoring for chondritis, and allowing the severely burned region to auto-amputate. This is especially true for large total body surface area (TBSA) burns such as the one sustained by the patient described, where large areas of excision and grafting must be done to save the patient €™s life. This is largely a problem of ear deformity, not function, and therefore has a lower priority.

Mafenide acetate (Sulfamylon) cream is the best topical antibiotic agent suitable for the ear because it penetrates the eschar and cartilage. It is applied twice daily. Trauma to the ear by dressings, endotracheal tapes, or pillows should be avoided. The eschar should not be completely debrided, as it acts as a biological dressing that prevents desiccation.

The more aggressive managements of costochondral reconstruction or retroauricular flap reconstruction are not warranted in the acute phase and can limit future reconstructive options.

Excision and full-thickness skin grafting can be considered in the intermediate management but are generally contraindicated in such a large burn after the wounds become heavily colonized. Excision and split-thickness skin grafting can be considered as well.

66
Q

A 19-year-old man who sustained burns to 40% of the total body surface area at 4 years of age comes to the burn clinic for consultation regarding correction of the scalp defect shown. Physical examination shows patchy alopecia of the temporal scalp and hypertrophic scarring. Which of the following is the most appropriate surgical procedure for reconstruction of this patient=s scalp?

(A) Hair transplantation with micrografts and minigrafts

(B) Orticochea flap

(C) Serial excision and closure

(D) Temporoparietooccipital (Juri) flap

(E) Tissue expansion

A

The correct response is Option E.

The patient shown has a large (>25 cm2) parietal scalp defect that resulted from a flame burn. It is clear that hair transplantation has already been attempted with minimal take and coverage. Hair transplantation with micrografts and minigrafts is a technique that has been described to treat large areas of burn alopecia. However, it is usually reserved for smaller areas, or as a revisional procedure to camouflage incisions that result from rotational flaps.

Tissue expansion is the preferred method for secondary reconstruction of a large parietal scalp defect. Approximately 50% of scalp can be reconstructed with expanded scalp tissue, although this may require multiple stages. Expander complication rates may be as high as 25% and may include infection, exposure, extrusion, and device failure. Scalp expansion in children is difficult. There is a higher incidence of infection and decreased tolerance of pain. Moreover, there is a risk of skull deformation after a prolonged period of expansion. Therefore, deferral until the patient reaches young adulthood is reasonable.

Direct closure in a staged manner with serial excision or as a single €‘stage procedure with rotation advancement flaps is appropriate for small to medium-sized defects of the anterior and parietal scalp. The size of this defect would preclude this therapy in this case.

Temporoparietooccipital flaps as described by Juri are optimally suited for reconstruction of large defects of the anterior scalp. Orticochea flaps are classically described for reconstruction of large defects of the occipital scalp. They are not useful for parietal defects because tissue advancement from the contralateral parietal scalp up over the vertex is often inadequate for defect closure.

67
Q

A 35-year-old man is brought to the emergency department after he sustained frostbite injuries to both hands and feet when he became lost while skiing and was exposed to subfreezing temperatures. Both hands and feet are rewarmed by immersion in circulating water for 30 minutes. Which of the following is the most appropriate diagnostic tool to establish the level of amputation in this patient?

A ) Contrast MRI

B ) Noncontrast CT

C ) Noncontrast MRI

D ) Technetium-99m bone scanning

E ) Three-view plain radiography

A

The correct response is Option D.

The initial management of frostbite injury is rapid rewarming by immersion of the involved areas in 40 °C to 42 °C circulating water for 15 to 30 minutes. The severity of the frostbite (first to fourth degree) should be noted. The wounds are dressed with loose, nonadherent dressings. Topical antibiotic ointment is applied and tetanus prophylaxis administered if indicated. Parts that are frostbitten should be splinted and elevated. The involved areas must be monitored for compartment syndrome.

Plain radiography, CT, and MRI are useful for delineating bone injuries, ligamentous injuries, and foreign material. However, the most useful tool for early prediction of the level of amputation is two-phase technetium-99m bone scanning. A retrospective study shows that an initial bone scan (as early as day 3) has excellent specificity in evaluating the severity of frostbite injury. Study data have shown a direct correlation between the demarcation zone of uptake in the phalanges and the eventual level of amputation (positive predictive value, 0.84). A second scan taken on approximately day 7 was more sensitive and informative. A strong correlation existed between positive uptake and eventual healing (negative predictive value, 0.99). This study showed that technetium-99m bone scanning, in the first few days after frostbite injury, indicates the level of amputation in severe frostbite in almost 90% of cases.

68
Q

A 26 €‘year €‘old man is brought to the emergency department 30 minutes after sustaining injuries in a motor vehicle collision. Examination shows full-thickness burns on 28% of the total body surface area, flail chest, pelvic hematoma, and an open fracture of the right ankle. Radiographs show a fracture of the pelvis. He is intubated and mechanically ventilated. A vena cava filter is placed. Open reduction and internal fixation of the ankle fracture is performed followed by debridement of the right lower leg. A free tissue transfer is planned to cover the exposed tibia, patella, and peripatellar tendon. To avoid flap thrombosis, which of the following is the most appropriate time to perform the tissue transfer?

(A) Immediately

(B) Two days after debridement

(C) Six days after debridement

(D) Two weeks after debridement

(E) Six weeks after debridement

A

The correct response is Option E.

Assuming that the patient described needs a free flap to provide coverage of the exposed knee structures, the optimal time for free tissue transfer in burn patients, following debridement, is four to six weeks after injury. A recently published series of burn patients who underwent free tissue transfer for limb salvage reported an overall flap loss rate of 12% (5/42). Five of the 14 flaps done between five days and six weeks were lost, compared to a success rate of 100% in 21 flaps performed after six weeks. The proposed mechanism for flap thrombosis was the hypercoagulable state associated with burn injury.

For nonthermal lower-extremity trauma requiring free tissue transfer, most studies support early flap reconstruction within the first week after injury. Both military and civilian injuries benefit from aggressive debridement and immediate or nearly immediate coverage.

Other possible options for reconstruction include subatmospheric sponge dressing, Integra, and staged skin grafting. The scenario described, however, mandates the use of free tissue transfer.

69
Q

A 4-year-old boy has full-thickness burns on 20% of the total body surface area. Which of the following end points is the most appropriate indicator of adequate fluid resuscitation in this patient?

(A) Cardiac index of 2.0 l/min/m2

(B) Mean arterial pressure of 60 mmHg

(C) Positive inspiratory pressure of 25 mmHg

(D) Pulmonary wedge pressure of 15 mmHg

(E) Urine output of 1.0 ml/kg/hr

A

The correct response is Option E.

Since the recognition of the phenomenon of burn shock, significant progress has been made in the survival rates of pediatric and juvenile patients. Currently, patients with all but the largest total body surface area (TBSA) injuries can be expected to survive when treated promptly. There are several very important conceptual factors in the resuscitation of pediatric patients with burns. Intravenous fluid resuscitations usually are required for patients with smaller burns (10% €“20%). Venous access in small children may be a difficult issue, and a saphenous vein cutdown or an interosseous line is an acceptable short-term alternative. Children have proportionally larger body surface areas (BSAs) than adults; TBSA burns must be estimated using the pediatric modifications in the Lund-Browder classification, which demonstrates the relatively larger head and small thigh in pediatric patients. This results in higher weight-based calculations for resection volume (nearly 6 ml/kg per percentage burn), leading some to advocate a BSA-based resuscitation in addition to the required maintenance infusion as described by the Galveston Shriners Hospital (Galveston, TX) pediatric formula. Other centers, such as the Shriners Burn Institute in Cincinnati, use the Parkland formula with the addition of a maintenance rate.

Recommended end points are also higher in children, with urine output closer to 1 ml/kg/hr being a more appropriate goal. Children approaching 50 kg are probably better served by adult resuscitation parameters (30-50 ml/hr urine output) and calculations. Another concern with this population is the modest hepatic glycogen reserves, which can be exhausted quickly and sometimes require the change from Ringer €™s lactate to dextrose 5% in Ringer €™s lactate to prevent life-threatening hypoglycemia. For this reason, AccuChecks every four to six hours should be routine during the hypermetabolic state, especially for patients with larger burns.

Pediatric resuscitation protocols are based on the following formula (H is height [cm], W is weight [kg]):

BSA = [87 (H + W) €‘ 2600]/10,000

Pediatric resuscitation protocols are as follows:

C Shriners Burn Institute (Cincinnati): ml/kg per percentage burn plus 1500 ml/m2 BSA

o First eight hours: Ringer €™s lactate with 50 mEq sodium bicarbonate per liter

o Second eight hours: Ringer €™s lactate

o Third eight hours: Ringer €™s lactate solution plus 12.5 g of 25% albumin solution per liter

C Galveston Shriners Hospital: 5000 ml/m2 TBSA burn plus 2000 ml/m2 BSA, using Ringer €™s lactate plus 12.5 g 25% albumin per liter plus D5W solution as needed for hypoglycemia

Hourly urine output is a well €‘established parameter for guiding fluid management. The rate of fluid administration should be titrated to a urine output of 0.5 ml/kg/hr or approximately 30 to 50 ml/hr in most adults and older children (>50 kg). In small children, the goal should be approximately 1 ml/kg/hr. Failure to meet these goals should be addressed with gentle upward corrections in the rate of fluid administration by approximately 25%.

Because of several factors, such as pulmonary vasoconstriction, the same interpretive problems are true for central venous pressure or pulmonary capillary wedge pressure measurements. Again, the overall clinical response and general trends in these numbers are much more useful for adjusting fluid administration or chemotherapy to support cardiac function than are values from isolated measurements. Catheter-based resuscitation tends to deliver higher-based volumes than the traditional methods but has not demonstrated an improvement in morbidity or mortality. Research indicates that increased crystalloid cannot restore cardiac preload to baseline during the period of burn shock.

70
Q

A 26-year-old man is brought to the emergency department 90 minutes after sustaining injuries in a house fire. The patient says he has a headache. Physical examination shows second-degree burns to the face and anterior chest equaling 10% of the total body surface area, singed nasal vibrissae, and carbonaceous sputum. Ventilation with 100% oxygen is started; SaO2 is 95%. Which of the following is the most appropriate next step in diagnosis?

(A) Fiberoptic bronchoscopy

(B) Measurement of carboxyhemoglobin level

(C) Radiograph of the chest

(D) Spiral CT

(E) Ventilation-perfusion lung scan

A

The correct response is Option A.

Inhalation injury is the most frequent cause of death in burn patients. Smoke inhalation in combination with cutaneous burns is fatal in 30% to 90% of patients. Prompt diagnosis and aggressive early therapy are paramount in reducing the morbidity and mortality associated with this injury.

The clinical diagnosis of inhalation injury has traditionally rested on a group of clinical observations including facial burns, singed nasal hair, and a history of injury within an enclosed space. As a group, they underestimate the true incidence of inhalation injury.

The current standard for diagnosis of inhalation injury in most major burn centers is fiberoptic bronchoscopy. Findings include the presence of soot, charring, mucosal necrosis, airway edema, and inflammation. Widespread use of this technique has led to a two €‘fold increase in the diagnosis of inhalation injury.

Patients with inhalation injury also have carbon monoxide poisoning levels greater than 10%. Levels greater than 30% can cause headache, nausea, and behavioral disturbances. Levels greater than 40% cause a pathognomonic cherry €‘red skin discoloration. Carbon monoxide levels in the patient described would be a poor indicator of injury because the patient received 100% oxygen for the past two hours. It takes one hour for levels to fall by one half while patients breathe 100% oxygen.

Clinical findings of hypoxia, crackles, wheezing, and rhonchi, and initial findings on radiographs of the chest are poor indicators of inhalation injury. It usually takes five to 10 days before findings of focal infiltrates or diffuse pulmonary edema are evident on radiographs.

A spiral CT is useful in determining the presence of a pulmonary embolism but plays no role in the acute diagnosis of inhalation injury.

A xenon ventilation-perfusion lung scan is the most definitive study for diagnosis of inhalation injury. It demonstrates areas of decreased alveolar gas washout, which identifies sites of small airway obstruction caused by edema or fibrin cast formation; however, the study is time-consuming. It requires transport of the patient from the emergency department or burn unit to the radiology department and is therefore less practical.

71
Q

A 34-year-old man sustained deep burns to the entire body surface area above the level of the waist one year ago. Initial management of the burns included excision and split-thickness skin grafting. During his recovery, release of burn scar contractures will most likely be needed in which of the following anatomic regions?

(A) Axilla

(B) Elbow

(C) Hand

(D) Neck

(E) Trunk

A

The correct response is Option D.

According to multicenter study data, neck contracture release is performed 24% of the time, followed by axilla (20%), elbow (11%), trunk (11%), knee (10%), and hand (9%). Release procedures are often performed in more than one site. The most common factors prompting contracture release are decreased range of motion or impaired function, unsatisfactory cosmetic appearance, pain, and pruritus.

Neck contractures produce significant tightening of the lower face and mechanical disability. At times, the contracture is so severe that intubation may be difficult. In such cases, the neck must be released under local anesthesia with sedation before intubation and general anesthesia. Release of scar, which must be complete and often involves the platysma, may involve Z €‘plasty, skin grafting, or tissue-expanded skin in adjacent, unburned skin. Postoperatively, the patient is fitted with a neck splint to extend the neck to prevent contraction and provide pressure necessary for optimal scarring.

72
Q

A 14 €‘year €‘old boy is brought to the office because he is unable to extend the proximal interphalangeal (PIP) joint of the long finger of the dominant right hand beyond 60 degrees of flexion. Six months ago, he sustained partial-thickness burns to the palmar surface of the long finger. The remaining fingers were spared. He has been performing active and passive range of motion exercises under the guidance of a therapist. Examination shows a thick scar extending the full width of the palmar skin and from the mid proximal phalanx to the distal interphalangeal flexion crease. With full passive flexion of the metacarpophalangeal (MCP) joint, the PIP joint can be extended nearly completely. Which of the following is the most appropriate management?

(A) Continued hand therapy and observation

(B) Daytime dynamic extension splinting and nighttime static extension splinting

(C) Injection of a corticosteroid into the scar and continued hand therapy

(D) Release of the contracture and full €‘thickness skin grafting

(E) Z €‘plasty lengthening of the scar and continuous static extension splinting

A

The correct response is Option D.

Despite optimal management of hand burns with splinting and occupational therapy, scar contractures may still occur. As in other contractures of the fingers, management depends on the elements responsible for the loss of motion. In the patient described, the severe PIP joint contracture shows adequate correction with MCP joint flexion, implying a primary derangement of the skin. Involvement of deeper structures (volar plate, collateral ligaments) would not correct with altered position of the MCP joint.

As a primary intervention, the optimal treatment of the patient described is division of the contracted cord and interpositional, full €‘thickness skin grafting.

For initial treatment and early, immature contracture management, therapy and dynamic splinting are critical interventions. In an established, mature burn scar, attempts at stretching the scar (continued hand therapy or dynamic extension splinting) will be unlikely to correct the defect.

Injection of a corticosteroid may be beneficial for keloids or hypertrophic scars; however, it would not correct the burn scar contracture in the patient described.

Correction of PIP joint contractures with more extensive dissection (joint release) requirements may devitalize tissues essential for skin graft take. In the scenario described and in the setting of secondary corrective surgery, local flaps such as the cross €‘finger flap may prove more useful.

Local tissue rearrangement (Z €‘plasty) is effective in narrow scars but will be difficult and likely ineffective in broad scars.

73
Q

A 37 €‘year €‘old man is brought to the emergency department because he has pain, paresthesia, and difficulty extending the fingers of the right hand one hour after he sustained a high-voltage electrical injury. Physical examination shows burns over 20% of the total body surface area with an entrance wound in the right forearm and an exit wound in the left lower abdomen. Which of the following is the most appropriate management?

(A) Decompressive fasciotomy of the right forearm

(B) Elevation and splinting of the right upper extremity

(C) Infusion of calcium gluconate

(D) Serial physical examinations of the right upper extremity over eight hours

(E) Tangential excision of burned tissue and coverage with allografts

A

The correct response is Option A.

Compartment pressures greater than 30 mmHg, worsening paresthesia, pain on passive range of motion, or pulselessness in an extremity mandate immediate decompression as part of acute resuscitation after electrical injury. Fasciotomies should also include decompression of the median and ulnar nerves at the level of the wrist, as well as decompression of the pronator quadratus, which often sustains significant collateral injury, given its proximity to bone, which has higher resistance than the soft tissue and generates greater heat than more superficial structures. Decompression of the ulnar nerve at the level of the cubital tunnel should also be considered.

Elevation and splinting are helpful adjuncts after burn injury to an extremity, but these measures should not delay the definitive management of compartment syndrome, which is fasciotomy.

Infusion of calcium gluconate is indicated after hydrofluoric acid chemical burns.

Although serial physical examination is a guiding diagnostic principle of surgery, this approach is contraindicated in a patient who meets criteria for compartment syndrome and requires fasciotomy.

Although wide excision of burned tissue is desirable, it is often difficult to appreciate the full magnitude of tissue loss, arguing against immediate excision and definitive coverage. Staged excision, combined with temporary coverage, is acceptable after distal perfusion has been restored.

74
Q

Which of the following is most appropriate in preparation of deep partial €‘thickness burn injuries to the hand before skin grafting?

(A) Application of topical enzymatic debriding agents

(B) Delay of surgical intervention until natural separation of eschar occurs

(C) Excision of the skin to the viable subcutaneous fat

(D) Tangential excision until pinpoint bleeding occurs

(E) Whirlpool hydrotherapy until indeterminate areas are declared

A

The correct response is Option D.

The best treatment of eschar in deep partial thickness or indeterminate burn injuries is early tangential excision until pinpoint bleeding occurs.

Proper diagnosis, evaluation, stabilization, and resuscitation are required in every case. In patients with multiple traumatic injuries, other injuries must be assessed and treated before excision of the primary burn. After it has been established that no contraindications to immediate primary excision exist, the patient is taken to the operating room, where all burn excisions, tangential or full thickness, are performed with the patient under general anesthesia administered by personnel experienced in burn injuries. Areas of obvious full-thickness burn are excised to the level of viable fat or fascia using electrocautery.

Delay of surgical intervention until natural eschar separation has been shown to significantly increase mortality, increase hospitalization, increase incidence of burn wound sepsis, and also contribute to burn scar hypertrophy and contracture.

Early surgical excision into viable subcutaneous fat is not appropriate for partial €‘thickness or indeterminate burns. It is, however, appropriate management of obvious full €‘thickness burns.

Deep partial €‘thickness eschars, and also areas of indeterminate burn, are treated by tangential excision using a Goulian knife. Tangential excision continues in these areas until pinpoint bleeding is seen. The points of bleeding are close together in the upper layers of the dermis, known as the papillary dermis. Burn wounds at this level are partial-thickness injuries that heal adequately with a biologic barrier material. If tangential excision proceeds to the reticular dermis, which is in the deeper dermal layers, bleeding points become more widely separated. At these deeper levels of tangential excision, the yellow hue of subcutaneous fat may become visible. Tangential excision to this deeper layer is best treated with a biologic barrier material followed by skin grafting.

Application of topical enzymatic debriding agents and whirlpool hydrotherapy are inappropriate for management of burn eschar.

75
Q

A previously healthy 55-year €‘old man is brought to the emergency department three hours after sustaining partial-thickness scald burns to the arms, torso, and legs, comprising 35% of the total body surface area. He has smoked one half pack of cigarettes daily for the past 20 years. The patient is breathing comfortably on room air and oxygen saturation is 99%. On physical examination, he is alert, calm, and cooperative. The patient is admitted to the hospital for wound care. Three days after the injury, respiratory distress develops and he is intubated. Which of the following is the most likely mechanism of respiratory failure in this patient?

(A) Aspiration pneumonia

(B) Inflammatory interstitial edema

(C) Inhalation injury

(D) Pulmonary embolism

(E) Severe chronic obstructive pulmonary disease

A

The correct response is Option B.

Burn injury results in local production of immunoregulatory mediators, which results in the production of prostaglandins, thromboxane A2, and reactive oxygen metabolites. These vasoactive inflammatory mediators result in edema and fluid extravasation from the vascular system. Burns of more than 20% or 30% of the total body surface area result in significant distant soft tissue edema and microvasacular injury. In the lungs, interstitial edema results in decreased oxygenation and may progress to respiratory distress syndrome. Gram-negative pneumonia is common among critically ill burn patients, especially intubated patients.

Aspiration is not a likely source in an alert patient. Smoking is likely to have contributed to this patient=s susceptibility to lung disease, although the primary, overwhelming factor involved is the inflammatory response to a critical burn injury. His normal breathing and oxygen saturation on admission rule out a severe underlying pulmonary process. Inhalation injury is common in fires but not scald injury. Pulmonary embolism may result after prolonged immobilization but is much less likely the source of respiratory failure and pneumonia in the acute burn patient.

76
Q

A 43 €‘year €‘old man who works as an electrical lineman is brought to the emergency department one hour after he sustained a high-voltage electrical injury. Physical examination shows a 1.5 x 2-cm area of eschar on the palmar aspect of the right hand and an area of charred tissue on the right foot that is presumed to be the exit wound. Pulses in all extremities are within normal limits. Evaluation of the hand shows absence of sensation and muscle weakness. Which of the following diagnostic studies is most appropriate to evaluate the extent of muscle damage in the patient €™s hand?

(A) Angiography

(B) CT

(C) Electromyography

(D) MRI

(E) Technetium-99m scanning

A

The correct response is Option D.

High-voltage electrical injuries (greater than 1000 volts) to the extremities present a difficult diagnostic dilemma. Often, the injury is significantly more extensive than indicated by the cutaneous examination. Muscle necrosis may be present under normal skin. Electrical injuries often involve deep structures and subfascial edema may occur, leading to further muscle necrosis. Various studies have been used to evaluate the burned extremity when there is no need for urgent escharotomy. MRI is the most appropriate choice because it is the most sensitive of the tests listed. MR images do not demonstrate tissue edema.

In patients who have sustained high-voltage electrical injuries, observation is appropriate. In electrical burns, sensation often cannot be used as an adequate judge because paresthesia is common because of the low resistance of the nerve to electric current. Myoglobinuria is associated with significant muscle damage and may predict the need for fasciotomies.

77
Q

A 25-year-old firefighter is transferred to the burn unit for management of burns to the hands 10 days after he sustained burns to 85% of the total body surface area (TBSA). His condition is stable, and vital signs are within normal limits. He has undergone multiple excision and grafting procedures. Current physical examination shows large areas of exposed tendon over the dorsum of both hands after excision of the burned skin. Partial-thickness burns of adjacent tissue are noted. Because of the extent of this patient €™s TBSA burns, regional and free tissue flaps are precluded. The most appropriate intervention for wound coverage is application of which of the following?

(A) Allograft skin graft

(B) Dermal regeneration template (Integra)

(D) Expansion of adjacent tissue

(D) Porcine xenografts

(E) Temporary biosynthetic skin substitute (Biobrane)

A

The correct response is Option B.

The patient described requires wound coverage with dermal regeneration template (Integra). Exposure of the extensor tendons precludes the use of skin grafts and no local flaps are available in this patient €™s burned hands because of adjacent scarring. Allograft skin would have similar problems of adherence to the exposed tendons and would require overly epidermal autografts. Regional and free flaps are also unavailable.

The use of tissue expansion to increase the size of full €‘thickness grafts and to improve the availability of local flaps while decreasing donor site morbidity has been suggested. The patient described is not a candidate for this procedure because of the local partial-thickness burns. Even if the local tissue was uninjured and could be expanded, the presence of exposed tendons does not allow for the protracted time associated with expander reconstruction. Expanders in the upper extremity have also been associated with a high rate of complications such as infection and extrusion.

Temporary biosynthetic skin substitute (Biobrane) consists of a collagen gel dermal analog and a Silastic silicone rubber epidermal analog. It is indistinguishable from porcine xenograft in terms of pliability, wound adherence, and formation of granulation tissue. Both Biobrane and porcine xenograft may be used for coverage of partial-thickness burns. Both allow wound re-epithelialization, at which point the Biobrane or allograft can be peeled off.

Bilaminate skin substitutes such as Integra (collagen €‘glycosaminoglycan dermis/Silastic epidermis) and Alloderm regenerative tissue matrix (autologous €‘allogeneic dermis) plus a split-thickness skin graft emulate the desirable properties of dermis and epidermis, particularly with respect to pore size. The outer membrane requires a small pore size to act as a barrier to microorganisms while allowing water vapor permeability. The inner layer requires a larger pore size to permit ingrowth of granulation tissue from the wound bed. Use of Integra and Alloderm for full €‘thickness wound coverage has been associated with less scarring than with conventional skin grafts. This decreased scarring may in turn minimize secondary contracture and improve outcome, particularly in the hand. Both require thin epidermal grafts for coverage, though only Alloderm is able to support immediate skin grafting.

78
Q

A 26-year-old man is brought to the burn unit after sustaining severe thermal burns and inhalation injury. Increase in which of the following elements on peripheral blood smear is most predictive of a fatal outcome in this patient?
(A) C-reactive protein
(B) Erythroblasts
(C) Erythrocyte sedimentation rate
(D) Hemoglobin
(E) Leukocyte count

A

The correct response is Option B.

A recent study from Germany studied the prognostic significance of erythroblasts in peripheral blood and their correlation with a fatal outcome from a burn injury. Erythroblasts are not normally present in peripheral blood, except in the fetus and the newborn. After that stage, they may be associated with severe disease states such as cancer, infection, and severe hematopoietic diseases. According to the study, the odds ratio estimate for the presence of erythroblasts and fatal outcome was 8.2. This is greater than the ratio for any other marker, including standard markers of age, sex, percentage of total body surface area burned, total area of third-degree burn, inhalation trauma, white blood cell count, C-reactive protein level, and hemoglobin level.

79
Q

A 26-year-old man who works at a glass factory comes to the emergency department 30 minutes after both hands and forearms were exposed to hydrofluoric acid in a chemical accident. Immediately after the injury, the hands were wrapped in gauze. Examination of the hands and upper extremities shows erythema and severe pain. Which of the following is the most appropriate next step in management?
(A) Application of topical polyethylene glycol solution to the burned areas
(B) Lavage of the burned areas with sodium bicarbonate solution
(C) Subcutaneous injection of calcium gluconate into the burned areas
(D) Intravenous infusion of sodium bicarbonate
(E) Excision of burned tissue to the level of the subcutaneous fascia

A

The correct response is Option C.

The best treatment for this patient with hydrofluoric acid topical burns is calcium gluconate injection. Hydrofluoric acid, which is commonly used in industrial occupations, produces burn when it comes into contact with skin. Initial treatment of lavage with copious amounts of water reduces the amount of acid in the skin. Irrigation with bicarbonate solution is unnecessary and not readily available. Hydrofluoric acid exposure causes soluble fluoride ions to traverse the skin into the subcutaneous tissue, where it results in tissue liquefaction necrosis. Fluoride ions also bind magnesium and calcium, resulting in hypomagnesemia and hypocalcemia. Appropriate treatment would consist of subcutaneous or arterial infiltration (into the burned site) of calcium or magnesium gluconate to inactivate the fluoride ions. This binding of fluoride ions immediately reduces pain in the site and can be used as an indication of appropriate therapeutic levels. Intravenous sodium bicarbonate is not indicated in hydrofluoric acid burns. Topical polyethylene glycol solution application would not be appropriate because it has not been demonstrated to be effective in this type of chemical burn. Excision is unnecessary as a first-line treatment.

80
Q

A 35-year-old man is brought to the emergency department because of deep burn injury to the dorsum of the index, middle, and ring fingers of the dominant left hand (shown) resulting from direct contact with a hot motorcycle engine during a motor vehicle collision. On tangential excision, tendons are exposed in each finger and loss of the paratenon is noted. Which of the following is the most appropriate surgical management?

(A) Split-thickness skin grafting
(B) Dorsal metacarpal artery flaps
(C) Cross-finger flaps
(D) Reverse radial forearm flap
(E) Groin flap

A

The correct response is Option D.

Because the paratenon has been removed, immediate split-thickness skin grafting will result in areas of graft loss. Dressing changes could be performed followed by grafting on a granulated bed. This method of treatment risks tendon desiccation while awaiting granulation tissue.

Dorsal metacarpal artery flaps can be useful in reconstructing dorsal hand and proximal finger defects. To repair a dorsal finger defect more distally, a reverse dorsal metacarpal artery flap could be used.

Cross-finger flaps use dorsal skin of adjacent fingers to resurface palmar defects. A variation of the cross-finger flap can be used to resurface a dorsal defect. This requires de-epithelialization of the flap and then skin grafting of the deeper surface of the flap.

The groin flap is a pedicled flap based on either the superficial circumflex iliac artery or the superficial inferior epigastric artery that can resurface wounds of the upper extremity. It has the disadvantages of requiring two surgical stages and the hand remaining dependent during the initial period of flap attachment, which can encourage edema and stiffness. In addition, groin flaps are bulky and require revisional surgery on the dorsal hand.

A reverse radial forearm flap would be the best choice for resurfacing the dorsal hand and fingers because it is soft, thin, and compliant. This may be most safely accomplished in stages with syndactylization performed at a second stage. This flap, however, has been longitudinally split to simultaneously resurface two areas of the hand. The two main disadvantages of this flap include the sacrifice of a major artery and an unaesthetic and potentially unstable grafted donor site.

81
Q

A 35-year-old man is brought to the emergency department two hours after he sustained second- and third-degree burns to the anterior abdomen and chest, and lower extremities. Weight is 65 kg (143 lb). Which of the following is the most appropriate hourly volume of fluid resuscitation over the next six hours?
(A) 400 ml
(B) 800 ml
(C) 1200 ml
(D) 1600 ml
(E) 2000 ml

A

The correct response is Option C.

Initial fluid management of acute burns occurs over the first 24 hours after the burn injury. The total volume of fluids is determined using the Parkland formula (4 ml _ weight in kg _ % total body surface area [TBSA]). Half of the total volume is transfused in the first 8 hours and the rest in the next 16 hours.

In an adult, TBSA is determined by the “Rule of Nines,” as follows:

Head = 9%
Each upper extremity = 9%
Anterior thorax (chest + abdomen) = 18%
Posterior thorax (chest + abdomen) = 18%
Each lower extremity = 18%
Groin = 1%

In this case, the TBSA is 18% for the anterior thorax + (18% _ 2) for both lower extremities, for a total of 54%. Note that for purposes of the Parkland formula and resuscitation, only second- and third-degree burns are counted.

Therefore, in this patient, 4 ml _ 65 kg _ 54% TBSA = 14,040 ml total fluids over the first 24 hours. The patient requires 7020 ml of fluids in the first 8 hours. Because two hours have passed since the injury, the 7020 ml will be evenly divided and administered over the next 6 hours with an hourly volume of 1170 ml.

82
Q

A 10-year-old girl who sustained an electrical burn to the right upper extremity 10 days ago has undergone multiple debridements of the distal volar forearm since that time. The residual volar forearm wound is clean and measures 4 _ 8 cm. Full-thickness skin loss is noted, as well as segmental loss of the flexor carpi ulnaris tendon. The ulnar nerve is exposed but appears in continuity. The patient has sensory loss in the ulnar nerve distribution and no ulnar intrinsic hand function. Which of the following is the most appropriate management?
(A) Daily whirlpool therapy and dressing changes
(B) Full-thickness skin grafting
(C) Resection of the exposed ulnar nerve and sural nerve grafting
(D) Flap coverage
(E) Tendon transfers and flap coverage of the wound

A

The correct response is Option D.

This child’s wound requires flap coverage. Vascularized tissue in the form of local or distant flaps is recommended for coverage of exposed vital structures such as nerves, blood vessels, tendons, and bone. Therefore, wound care regimens alone and skin grafting are inadequate treatment options.

Early repair or grafting of the ulnar nerve is not indicated due to the inability to predict the amount of recovery at this early stage. Children also heal significantly better than adults, particularly in cases of nerve injury. For the same reason, tendon transfer is not indicated at this early juncture. However, appropriate splinting and therapy should be used while awaiting recovery of the nerve to prevent the claw deformity associated with low ulnar nerve palsy.

True electrical burns are completely different from thermal burns. Entrance and exit wounds can usually be identified and can be widely separated. The damage to underlying structures such as musculature and neurovascular structures typically far exceeds the visible cutaneous injury. Most patients will require decompression with fasciotomies and multiple debridements.

83
Q

A 25-year-old man who works as an electrician is brought to the emergency department by ambulance one hour after he fell from a ladder and grabbed a high-tension power line carrying approximately 50 kV with his right hand. Pulse rate is 120/min and blood pressure is 120/70 mmHg. Physical examination shows a burn mark over the right palm and a burn wound over the right scapula. No distal pulses are palpable. Which of the following is the most appropriate initial management of the wound to the upper extremity?
(A) Splinting of the hand and wrist in a position of function and elevation
(B) Topical application of mafenide and an occlusive dressing
(C) Escharotomy
(D) Fasciotomy
(E) Arterial grafting to the radial artery segment

A

The correct response is Option D.

Clinically, electrical injuries are divided into high- and low-tension injuries based on voltage. High-voltage injuries, as in this scenario, occur with contact of an electrical source greater than 1000 V. Electrical injuries of this magnitude usually result in significant myonecrosis as well as neural injury.

In the presence of such a significant injury, fasciotomy should be performed because continued muscle necrosis and swelling will certainly result in compartment syndrome. Decompression of the median nerve at the wrist should be ensured at the time of forearm fasciotomy. Other mainstays of treatment of electrical injuries to the upper extremity are repeated debridements of necrotic muscle and early flap coverage to decrease the incidence of amputation, infection, and renal failure. Escharotomy is the treatment for full-thickness thermal burns to the hand and arm. Mafenide acetate solution (Sulfamylon) would have efficacy for thermal burns in which penetration of cartilage is required. Arterial grafting may be required if there is evidence of a thrombosed radial artery, but this is not the most appropriate initial treatment. Splinting in a position of function is important but is not the most appropriate initial treatment.

84
Q

According to the American Burn Association guidelines, which of the following patients has a major burn injury requiring triage to a specialized burn center?

(A) 4-year-old boy with partial-thickness burn on the dorsal aspect of the forearm
(B) 16-year-old girl with partial-thickness burn over the shoulders and upper back
(C) 40-year-old man with type 1 diabetes mellitus and full-thickness perineal burn
(D) 55-year-old woman with full-thickness burns of the volar aspect of the right arm

A

The correct response is Option C.

The major burn criteria of the American Burn Association identify individuals, such as those with diabetes mellitus, who need specialized treatment because they are at high risk for postburn morbidity and mortality. The criteria for triage to a specialized burn center include:

$ second- or third-degree burn over more than 10% of the total body surface area (BSA) in a patient younger than 10 years or older than 50 years
$ second- or third-degree burn over more than 20% of the total BSA in a patient of any other age
$ significant burn of the face, hands, feet, genitalia, perineum, or skin over major joints
$ third-degree burn over more than 5% of the total BSA in a patient of any age
$ burn with concomitant inhalation injury, significant electrical injury including lightning strike, or significant chemical injury
$ burn with a significant preexisting medical disorder that complicates management, such as diabetes mellitus or heart disease
$ burn with concomitant trauma
$ burn in a patient who might have special social or emotional needs or require long-term support, as in a child who has been abused or neglected

85
Q

Topical silver sulfadiazine may produce which of the following sequelae?

(A) Carbonic anhydrase inhibition
(B) Granulocyte reduction
(C) Methemoglobinemia
(D) Staining of the skin on contact

A

The correct response is Option B.

Silver sulfadiazine (Silvadene) is a commonly used topical burn agent. It may result in granulocyte reduction (neutropenia and thrombocytopenia). Carbonic anhydrase inhibition may occur with mafenide acetate (Sulfamylon), resulting in metabolic acidosis. Silver nitrate is an excellent topical agent and has no gram-negative resistance; however, brown staining of skin and equipment is common and methemoglobinemia may rarely occur.

86
Q

A 30-year-old man sustains burns over 42% of the total body surface area and is resuscitated using the Parkland (Baxter) burn formula. Five percent albumin is added to the resuscitation fluid 24 hours after the injury to achieve which of the following?

(A) Maintenance of intravascular volume
(B) Normalization of intravascular pH
(C) Nutritional support
(D) Provision of coagulation cofactors

A

The correct response is Option A.

Acute burn resuscitation using the Parkland (Baxter) formula is based on the patient’s physiologic response to injury. Burns cause a leak in the capillary endothelium, which results in excessive protein loss. By 24 hours after the burn injury, the capillary leak is largely resolved. At that time, 5% albumin is added to the resuscitation fluid to help maintain intravascular volume.

87
Q

An 87-year-old man sustained a third-degree scald injury to the proximal forearm one hour ago. Early excision of the wound is performed, and the resultant 15 H 15-cm defect is covered using the dermal regeneration template (Integra). On which postoperative day should removal of the top silicone layer and placement of an autograft be performed?

(A) 1
(B) 7
(C) 14
(D) 21

A

The correct response is Option D.

Integra consists of a collagen-glycosaminoglycan layer covered by a silicone occlusive layer and can be used as a dermal substitute until it is replaced by the host’s own fibroblasts and endothelial cells. After the collagen-glycosaminoglycan layer is fully revascularized, the silicone layer can be removed and an autograft can be used for epidermal closure. This typically occurs at three weeks (on postoperative day 21) but may be done later, when full revascularization is evident.

Before postoperative day 21, revascularization is not adequate. In fact, the Moiemen, Staiano, and Ojeh study found that full revascularization was more likely to occur at four weeks.

88
Q

A 24-year-old man has a mentosternal contracture (shown above) one year after sustaining burns over 90% of the total body surface area. Which of the following reconstructive interventions is the most appropriate management of the contracture?

(A) Scar release and coverage with a dorsal scapular island flap
(B) Scar release and coverage with a free scapular flap
(C) Scar release and skin graft coverage with a thin split-thickness skin graft harvested from the scalp
(D) Scar release and use of the dermal regeneration template (Integra)

A

The correct response is Option D.

A mentosternal contracture usually requires a wide scar release and extensive tissue coverage. However, this patient who has burns over nearly the total body surface area has limited donor sites. Therefore, the dermal regeneration template should be used with thin split-thickness grafting. This reconstructive intervention provides an acceptably low rate of long-term recurrence of contracture and it uses available donor sites.

Scar release and coverage with a thin split-thickness skin graft have an unacceptable rate of recurrence of contracture. Scar release and coverage with a free scapular flap or dorsal scapular island flap are ideal options for reconstruction. However, unburned scapular skin is not likely to be available in a patient with burns over 90% of the total body surface area.

89
Q

A 45-year-old farmer has worsening ulceration of the right cornea two days after anhydrous ammonia was splashed in his eyes. Immediately after this accident, the patient’s eyes were washed with saline for 45 minutes. Which of the following is the most appropriate explanation for the worsening of this patient=s condition?

(A) Bacterial infection
(B) Heat from the examining light
(C) Liquefaction necrosis
(D) Scar tissue

A

The correct response is Option C.

Liquefaction necrosis is the most likely cause of the worsening eye injury. Because anhydrous ammonia is an alkaline solution, it can denature and dissolve proteins and lyse cell membranes. This increases the penetration of the alkaline solution into the eye, furthering the damage.

If a bacterial infection occurs, it usually begins more than two days after the injury. Examination of the eyes with a light is unlikely to cause further damage. Scar tissue is unlikely to develop within two days.

90
Q

A 35-year-old man is brought to the emergency department after sustaining burns covering 40% of the total body surface area (TBSA). Physical examination shows burns to the face and chest, facial edema, and singed nasal hairs. He has stridor; respirations are 24/min. With the patient receiving 100% oxygen, pulse oximetry is 98%; arterial blood gas analysis shows a carbon monoxide level of 30%. Which of the following is the most appropriate next step in management?

(A) Bronchoscopy
(B) Cricothyroidotomy
(C) Escharotomy of the chest
(D) Hyperbaric oxygen therapy
(E) Intubation

A

The correct response is Option E.

Smoke inhalation and potential carbon monoxide poisoning should be suspected in any patient who has sustained facial burns and has carbon deposits within the oropharynx, singeing of facial hair, or carbonaceous sputum. If these findings are present, 100% oxygen should be administered immediately using a non-rebreather mask. Because pulse oximetry cannot differentiate between oxyhemoglobin and carboxyhemoglobin, artificially high readings are often obtained even in patients with carbon monoxide toxicity. The half-life of carboxyhemoglobin at an inspired oxygen fraction (FIO2) of 0.21 is 240 minutes, but at an FIO2 of 1.0, the half-life improves to 30 minutes.

Any patient who has a carbon monoxide level of 30% or higher on arrival in the emergency department, even after receiving supplemental oxygen, has suffered severe toxicity. Intubation should be performed immediately because coma and respiratory depression may occur with levels of 40% to 50%, and death can occur at levels of 50% or higher. Patients who have dyspnea, stridor, tachypnea, and/or swelling of the tongue or oropharynx, or who are using accessory respiratory muscles, should also undergo immediate intubation. If progressive swelling of the airway is a concern during fluid resuscitation, intubation should be considered.

Hyperbaric oxygen may be administered to patients with severe carbon monoxide poisoning, but, in emergency cases, this time-consuming process is not advocated.

Bronchoscopy or ventilation scanning should be performed to confirm smoke inhalation, but is less urgent than management of the airway in this patient.

Cricothyroidotomy is indicated only in patients with severe swelling of the airway when intubation is not possible.

Chest escharotomy is indicated in patients with respiratory compromise and circumferential chest burns, but should not be performed before breathing has improved.

91
Q

In a patient who sustained burns over 35% of the total body surface area four hours ago, which of the following is the most important factor in development of shock?

(A) Dilation of the peripheral vasculature
(B) Hypovolemia
(C) Myocardial depression
(D) Paralytic ileus
(E) Renal failure

A

The correct response is Option B.

In an untreated major thermal burn, the most important factor in the initial development of shock is hypovolemia. Thermal injury disrupts capillary endothelial integrity and alters membranes. In a major burn, these actions occur even in unburned tissue because of circulatory and microcirculatory dysfunction. They lead to plasma leakage from the circulation, which results in decreased plasma volume, cardiac output, and urine output and increased systemic vascular resistance. Local and systemic inflammatory mediators may play a role in these processes.

Dilation of the peripheral vasculature can cause shock later if sepsis occurs but is not a factor in the initial development of shock. Myocardial depression can develop later in shock if a systemic inflammatory response, severe hypovolemia, or sepsis occurs. However, it is not a factor initially. Paralytic ileus is a systemic result of a major burn but is not a factor in the initial development of shock. Renal shutdown is an effect of initial shock, not a factor in its development, but can occur from sustained hypovolemia.

92
Q

A 55-year-old man who weighs 90 kg (198 lb) is brought to the emergency department eight hours after sustaining first-degree burns to the head and neck and second- and third-degree burns to the entire anterior trunk and both lower extremities. According to the Parkland formula, the most appropriate management is fluid resuscitation with administration of lactated Ringer’s solution for the next eight hours at a rate of how many milliliters per hour?

(A) 607 mL/hr
(B) 709 mL/hr
(C) 1215 mL/hr
(D) 1823 mL/hr
(E) 2127 mL/hr

A

The correct response is Option D.

In a patient who has a second- and/or third-degree burn injury that covers more than 20% of the total body surface area (TBSA), acute fluid resuscitation should be performed with administration of lactated Ringer’s solution during the initial 24 hours after injury. The Parkland formula is used to estimate the amount of fluid required. According to this formula, lactated Ringer’s solution (4 mL/kg/% TBSA burned) should be administered during the first 24 hours. A total of 50% of the solution should be administered during the first eight-hour period and the remaining 50% over the next 16 hours.

The TBSA involved in a burn can be calculated using the “rule of nines.” According to this rule, the anterior trunk, the posterior trunk, and each lower extremity are assigned values of 18%. Each upper extremity and the head have values of 9%, and the neck has a value of 1%. In this patient, the burn of the anterior trunk is assigned a value of 18%, and each lower extremity burn is 18%, for a TBSA burn of 54%. First-degree burns, such as those of the head and neck, are not included in the TBSA calculation.

A 90-kg patient who has burns involving 54% TBSA will require 19,440 mL of fluid during the first 24 hours: 9720 mL during the first eight hours and 4860 mL in both the second and third eight-hour periods. Because he received no fluid during the first eight hours immediately after injury, 14,580 mL of lactated Ringer’s solution (9720 mL + 4860 mL) should be administered over the next eight hours to adequately resuscitate this patient. Divided into eight-hour totals, the solution is infused at a rate of 1823 mL/hr.

93
Q

A 38-year-old man sustained frostbite of the right hand four days ago. Examination shows necrosis distal to the metacarpophalangeal (MP) joints. Three-phase bone scanning shows viability of the proximal phalanx of each finger. Which of the following is the most appropriate management?

(A) Amputation at the level of the distal interphalangeal joints and primary closure
(B) Amputation at the level of the MP joints and primary closure
(C) Amputation at the level of the MP joints and radial forearm flap reconstruction
(D) Amputation at the level of the proximal interphalangeal (PIP) joints and groin flap reconstruction
(E) Amputation at the level of the PIP joints and second metacarpal artery flap reconstruction

A

The correct response is Option D.

Because frostbite injuries can cause devastating loss of tissue, aggressive management is often needed to salvage and provide vascularized coverage over viable bone. The level of skin loss does not always correlate with the level of bone viability. A three-phase bone scan can be used to determine at which level the bone is viable. Amputation of nonviable bone is performed along with amputation of the nonviable soft tissue. To salvage the bone, well-vascularized coverage is required, with either a pedicle flap or a free flap.

Amputation at the level of the distal interphalangeal joints and primary closure would leave the middle phalanx nonvascularized. Amputating at the MP joint would sacrifice viable bone. Amputation at the level of the MP joints and radial forearm flap reconstruction would sacrifice viable bone, and the radial forearm flap would not be needed for coverage. Amputation at the level of the PIP joints and second metacarpal artery flap reconstruction is inappropriate because a second metacarpal artery flap would be of insufficient length to cover the open area.

94
Q

Which of the following is a physiologic manifestation of shock following acute burn injury?

(A) Decreased cardiac output
(B) Decreased systemic vascular resistance
(C) Increased peripheral blood flow
(D) Increased plasma volume
(E) Increased urine output

A

The correct response is Option A.

Patients with acute burn shock exhibit hemodynamic changes similar to those seen with hypovolemic shock. Management should focus on volume resuscitation and maintenance of tissue perfusion.

Following burn injury, cardiac output is decreased to 40% to 60% of normal as a result of decreased plasma volume and increased systemic vascular resistance. The release of myocardial depressants further diminishes cardiac output. In addition, angiotensin II, catecholamines, neuropeptide Y, and vasopressin all act on arterial smooth muscle to cause vasoconstriction.

Decreased plasma volume and capillary pressure occur as a result of factors released from both injured and uninjured tissue and excessive fluid shifting that occurs within the first hour after injury. Although the total body water level remains constant, there is an increase in cell water content of 70% to 80%. Volume resuscitation is required to maintain tissue perfusion; unfortunately, this will also exacerbate burn wound edema.

Decreased urine output is caused by inadequate fluid resuscitation and occurs as a response to decreased tissue perfusion and increased systemic vascular resistance. Aggressive volume replacement is required during the first 24 to 48 hours after injury to restore intravascular fluid.

95
Q

A 26-year-old man who weighs 80 kg (176 lb) is brought to the emergency department three hours after sustaining superficial burns involving 20% total body surface area (TBSA), partial-thickness burns involving 15% TBSA, and full-thickness burns involving 25% TBSA. According to the Parkland formula, which of the following is the most appropriate method of fluid resuscitation for this patient?

(A) Administration of lactated Ringer’s solution 800 mL/hr for the next eight hours
(B) Administration of lactated Ringer’s solution 1300 mL/hr for the next five hours
(C) Administration of hypertonic saline solution 1000 mL/hr for the next 12 hours
(D) Administration of hypertonic saline solution 1500 mL/hr for the next five hours
(E) Administration of hypertonic saline solution 600 mL/hr for the next eight hours

A

The correct response is Option B.

In a patient who has second- and/or third-degree burn injuries that cover more than 20% of the total body surface area (TBSA), acute fluid resuscitation should be performed with administration of lactated Ringer’s solution during the initial 24 hours after injury. The Parkland formula is used to estimate the amount of fluid required. According to this formula, lactated Ringer’s solution 4 mL/kg/% TBSA burned should be administered during the first 24 hours. A total of 50% of the solution should be administered during the first eight-hour period and the remaining 50% over the next 16 hours.

An 80-kg patient who has burns involving 40% TBSA (second-degree burns involving 15% TBSA and third-degree burns involving 25% TBSA) will require 12,800 mL of fluid during the first 24 hours: 6400 mL during the first eight hours and 3200 mL in both the second and third eight-hour periods. Because he received no fluid during the first three hours immediately following injury, 6400 mL of lactated Ringer’s solution, or 1280 mL/hr, should be administered over the next five hours in order to adequately resuscitate the patient.

Vital signs and urine output should also be monitored in this patient to determine the success of resuscitation efforts.

Administration of hypertonic saline solution for fluid resuscitation in burn patients is controversial and is not universally recommended.

96
Q

Silver sulfadiazine is administered topically to a patient with severe burns of the trunk and upper extremities. Which of the following is the most likely adverse effect?

(A) Hyponatremia
(B) Metabolic acidosis
(C) Metabolic alkalosis
(D) Neutropenia
(E) Pain with application

A

D Neutropenia

97
Q

A 24-year-old woman has been hospitalized for the past week because she has toxic epidermal necrolysis syndrome involving 90% of total body surface area (TBSA) with sparing of the lower legs and feet. She has been mechanically ventilated, and her hands have been splinted in a functional position except during passive occupational therapy. Following discontinuation of the ventilation, this patient’s hands should be splinted for how long to prevent contractures?

(A) 6 weeks
(B) 3 months
(C) 6 months
(D) 1 year
(E) No additional splinting is required

A

The correct response is Option E.

Toxic epidermal necrolysis syndrome is an uncommon systemic condition characterized by acute inflammation of the skin and mucous membranes and sometimes the bowel and respiratory epithelium. In patients who develop this life-threatening condition, the mortality rate ranges from 20% to 60%. Although it is thought to be caused by an adverse reaction to an antibiotic, anti-inflammatory agent, or anticonvulsant, other factors, including viral, bacterial, and fungal infections and neoplasms, have also been implicated.

Patients with toxic epidermal necrolysis syndrome have sloughing of the skin at the dermal-epidermal junction. However, because injury occurs only to the level of the superficial dermis, the skin will heal without contracture. Therefore, no additional splinting is required following discontinuation of ventilation. Long-term sequelae of this condition include complications involving the eyes and fingernails and changes in skin pigmentation.

98
Q

An otherwise healthy 25-year-old man sustains second- and third-degree burns over 60% of total body surface area (TBSA). Which of the following immunologic responses is LEAST likely in this patient?

(A) Activation of helper T lymphocytes
(B) Increased circulating immunoglobulin G (IgG) level
(C) Increased interleukin-7 (IL-7) level
(D) Redistribution of T lymphocytes in the peripheral blood
(E) Suppression of circulating T lymphocytes

A

The correct response is Option B.

One immunologic response anticipated in this 25-year-old man who has sustained a 60% TBSA burn is impairment of cell-mediated immunity resulting from impairment of T-lymphocyte function. This manifests in burn patients as a delay in allograft rejection, suppression of the graft-versus-host response, and development of skin hypersensitivity reactions. Alterations in T-lymphocyte function include overall suppression of circulating T lymphocytes and a redistribution of T lymphocytes within peripheral blood and tissue compartments.

The initial response to significant burn injury is activation of helper T lymphocytes. B lymphocytes undergo functional changes resulting from the profound decrease in immunoglobulin levels following thermal injury. Although all classes of immunoglobulins are decreased, immunoglobulin G (IgG) levels exhibit the greatest decrease. This decrease can be attributed to plasma leakage, increased protein turnover, and decreased synthesis of IgG by B lymphocytes.

Interleukin-7 levels are increased during the first week after injury; this results in a decrease in the proliferative capacity of B lymphocytes.

99
Q

A 26-year-old man is brought to the emergency department with significant pain in the forearms 90 minutes after spilling hydrofluoric acid on his forearms at work. Immediately after the accident, the affected areas were copiously irrigated with water.

The most appropriate initial management is topical administration of which of the following agents?

(A) Calcium gluconate gel
(B) Mineral oil
(C) Polyethylene glycol
(D) Silver nitrate
(E) Silver sulfadiazine

A

The correct response is Option A.

This patient has severe pain after sustaining chemical burn injuries from a hydrofluoric acid spill. Hydrofluoric acid is a strong chemical agent used primarily in industries such as electronics manufacturing and glassworks but also found in rust remover and other household items. The mechanism of action of this chemical involves binding of fluoride ions to intracellular calcium and magnesium. The fluoride ions leech calcium and magnesium from the bones, causing cellular death and producing severe pain that may not develop until several hours after initial exposure. In addition, the cellular death leads to the release of potassium, resulting in hypocalcemia, hypomagnesemia, and hyperkalemia. Any patient who has sustained hydrofluoric acid burns should be hospitalized and monitored closely because of the risk for arrhythmia associated with these electrolyte imbalances.

The most appropriate initial management is copious irrigation of the involved areas with water or saline followed by administration of calcium gluconate. This agent, which neutralizes the hydrofluoric acid by binding to the affected fluoride ions, can be applied topically as a gel, infused intra-arterially, or infiltrated beneath the burn eschar. Because relief of pain is a primary goal of treatment, analgesic and anesthetic agents should not be administered, as the patient will be unable to judge the pain-relieving effect of the calcium gluconate.

Application of mineral oil is appropriate for patients with phenol burns. Irrigation with water is contraindicated in these patients because it can dilute the phenol, resulting in deeper penetration and increased tissue damage. Creosol burns should be treated with topical application of polyethylene glycol. Silver sulfadiazine is used in the treatment of thermal burns involving disruption of the skin and soft tissues. Silver nitrate is effective for cauterization of pyogenic granulomas.

100
Q

A 30-year-old man is brought to the emergency department after sustaining second-degree burns of the trunk involving 10% total body surface area (TBSA). The burns are cleansed, and several blisters are debrided. In order to provide antimicrobial activity, which of the following dressings should be applied to the wounds?

(A) Fibronectin-coated skin substitute (Transcyte)
(B) Porous collagen-glycosaminoglycan membrane (Integra)
(C) Silicone membrane-nylon fabric composite (Biobrane)
(D) Silver-coated wound dressing (Acticoat)

A

The correct response is Option D.

Treatment of partial-thickness burns can be accomplished through cleansing of the burn and application of either an antimicrobial or occlusive dressing. Most patients are treated with silver sulfadiazine (Silvadene); however, if a sulfa allergy is present, bacitracin, polymyxin/bacitracin (Polysporin), or mupirocin (Bactroban) can be used. Acticoat is a dressing material coated with a thin soluble layer of silver ion; it reportedly provides antimicrobial activity for as long as five days. The greatest advantage is a decrease in the number of dressing changes, with a subsequent decrease in pain, as well as decreased cost.

Biobrane and Transcyte are occlusive dressings that can be used for management of clean second-degree burns as long as they are applied within the first 24 hours. These dressings do not provide antimicrobial activity. Biobrane consists of a nylon fabric containing chemically bound collagen that is partially imbedded in a silicone film. As blood and serum clot within the nylon fabric, it adheres to the wound until epithelialization occurs, and then it sloughs. Transcyte consists of cultured human dermal fibroblasts on a semipermeable membrane bonded to nylon mesh. The mesh allows for growth of the dermal tissue, and the membrane forms a synthetic epidermis.

Integra is a bioengineered dermal substitute consisting of a bilayered membrane system. It is used for skin replacement after debridement of deep partial-thickness or full-thickness burns. The dermal replacement layer comes from bovine tendon cartilage, and the epidermal replacement is a synthetic silicone polymer that is removed following degradation of the dermal layer. A thin skin graft is then placed on the “neodermis.”

101
Q

Administration of anti-inflammatory agents to patients who have sustained frostbite is most likely to result in which of the following beneficial effects?

(A) Decreased production of prostaglandin I2
(B) Decreased production of thromboxane B2
(C) Increased production of prostaglandin F2a
(D) Increased production of prostaglandin I2

A

The correct response is Option B.

Based on theories regarding the pathophysiology of frostbite injury, adverse changes that occur within the microvasculature at the site of injury are thought to be caused by inflammatory mediators. The prostaglandins thromboxane B2 and prostaglandin F2a are thought to induce microvascular thrombosis because they cause platelet aggregation and vasoconstriction. In contrast, the prostaglandins I2 and E2 have antiplatelet activity, resulting in vasodilation. It is thought that the frostbite injury increases production of thromboxane B2 and decreases production of prostaglandin I2, resulting in an imbalance in favor of microvascular thrombosis. Experimental studies have shown increased levels of these mediators within the frostbitten tissue in both experimental animal models and in the blister fluid of frostbite patients. In addition, studies have demonstrated that specific thromboxane inhibitors can increase the survival of threatened tissue.

Because anti-inflammatory agents, such as aspirin and ibuprofen, inhibit cyclooxygenases, administration of these agents will decrease production of thromboxane B2 and prostaglandin F2a and thus block their harmful effects. However, an adverse effect of these agents is their inhibition of prostaglandin I2 and prostaglandin E2 production, which limits their protective effect.

102
Q

Monoclonal antibodies have been shown to limit the depth of burn injury by inhibiting neutrophil adhesion in which of the following zones?

(A) Zone of adherence
(B) Zone of coagulation
(C) Zone of hyperemia
(D) Zone of stasis

A

The correct response is Option D.

Patients with burn injuries have destruction of tissue proportionate to the length of time that the tissue has been exposed to the heat source and the temperature at the surface of the skin. There are three zones that can be described to delineate the pathophysiology of burn injury.

The zone of coagulation is that area of the skin that is exposed to the highest temperature, resulting in irreversible, uniform necrosis of cells. This zone involves the burn eschar and extends downward.

The zone of stasis surrounds the zone of coagulation; in this area, the cells sustain less direct injury initially. Instead, progressive injury occurs following the development of ischemia and subsequent impairment of blood flow. This zone is characterized by the formation of microthrombi within platelets, endothelial swelling, neutrophil adherence, deposition of fibrin, and vasoconstriction, leading to eventual cell death. However, because injury in this zone is potentially reversible, there have been numerous experiments involving the zone of stasis in an attempt to limit burn depth. Several experimental animal studies have shown that antibodies directed to receptors on neutrophils can block their adherence to vessel walls, preventing microvascular occlusion and leukocyte-mediated endothelial injury. The total burn surface area is subsequently decreased.

In the zone of hyperemia, there is vasodilation and increased blood flow caused by vasoactive mediators. Cellular injury in this zone is minimal and is completely reversible.

The zone of adherence describes anatomic regions within the body in which skin and subcutaneous tissue are connected to the underlying fascia. It is important for the surgeon to recognize these zones when planning and performing suction lipectomy, as excessive suctioning may result in contour deformities.

103
Q

A 40-year-old man sustains deep partial-thickness and full-thickness burns over 45% total body surface area (TBSA). Following fluid resuscitation for 24 hours, his temperature is 38.9%C (102.1%F), pulse rate is 120 bpm, respirations are 24/min, and blood pressure is 105/60 mmHg. Serum leukocyte count is 18,000/mm3 and urine output is 70 mL/hr. Which of the following is the most likely cause of these findings?

(A) Bronchopneumonia
(B) Burn wound sepsis
(C) Inadequate fluid resuscitation
(D) Inadequate pain control
(E) Systemic inflammatory response

A

The correct response is Option E.

This 40-year-old man with burn injuries has findings consistent with systemic inflammatory response syndrome (SIRS), an inflammatory condition that can be caused by soft-tissue trauma, bacteremia, sepsis, ischemia, or pancreatitis. SIRS typically occurs in patients who have burns of more than 30% total body surface area (TBSA); the cell damage caused by the burn often incites the inflammatory reaction. Hyperactivity of the immune system causes alterations in the metabolic, cardiovascular, gastrointestinal, and coagulation systems. Affected patients have hypermetabolism and exhibit increased cellular, endothelial, and epithelial permeability and microthrombosis.

The diagnosis of SIRS can be made in any patient with burn injury who meets at least two of the following criteria, occurring as a sudden alteration above baseline levels in the absence of any other condition:

  1. Body temperature lower than 36%C (96.8%F) or higher than 38.5%C (101.5%F)
  2. Heart rate greater than 90 bpm
  3. Respirations greater than 20/min, or carbon dioxide partial pressure (pCO2) less than 32 mmHg
  4. Serum leukocyte count less than 4000/_L, greater than 12,000/_L, or containing more than 10% band forms

Although the precise mechanism that causes SIRS is unknown, the humoral or cellular immune system activates the onset of symptoms. Because the complement system is typically activated in burn patients, this may interact with the coagulation, fibrinolysis, and kallikrein-kinin systems to trigger the onset of the syndrome. Other critical mediators include heat shock proteins, tumor necrosis factor-alpha, the interleukins IL-1, IL-6, and IL-8, and endotoxin.

Bronchopneumonia would be unlikely in a patient who sustained burn injuries only 24 hours ago and more often occurs in the second week following injury. Burn wound sepsis occurs when proliferating microorganisms exceed 105 per gram of tissue and can be characterized by fever, tachycardia, and leukocytosis, but again this would not appear within the first 24 hours after injury. Inadequate fluid resuscitation is unlikely in a patient with adequate urinary output. Inadequate pain control would cause tachypnea and tachycardia, but not fever or leukocytosis.

104
Q

To minimize the risk for hypertrophic scar formation and subsequent skin contractures in a patient who has sustained partial-thickness burns of the neck, attempts at healing by second intention should be limited to a maximum of how many weeks?

(A) 1
(B) 2
(C) 3
(D) 4
(E) 6

A

The correct response is Option C.

To minimize the risk for development of hypertrophic scars and subsequent skin contractures in a patient who has sustained partial-thickness burns of the neck, the wound should not remain open for more than three weeks. According to the results of one study, hypertrophic scars formed in 33% of patients whose wounds healed within three weeks, compared with 78% of patients whose wounds were left open for more than 21 days.

Burn scar contractures of the neck can be released using Z-plasty, local flaps, or thick split-thickness or full-thickness grafts. The surgeon may need to release the platysma with the scar in order to restore full extension. Long-term postoperative splinting and compression are essential for graft take. Tissue expansion of unburned adjacent skin is another alternative for resurfacing the burned area.

105
Q

A 25-year-old laborer sustains a burn of the dorsal aspect of the dominant right hand in a fire. Physical examination shows a deep partial-thickness burn that involves the entire dorsal aspect of the hand. Which of the following is the most appropriate management?

(A) Early excision of the burn wound and split-thickness skin grafting
(B) Coverage of the burn wound with a silicone membrane-nylon fabric composite (Biobrane)
(C) Aggressive topical wound care and occupational therapy
(D) Excision of the burn wound and coverage with a groin flap
(E) Excision of the burn wound and delayed split-thickness skin grafting

A

The correct response is Option A.

In this patient who has sustained a deep partial-thickness burn of the hand, the most appropriate management is early excision of the burn eschar followed by split-thickness skin grafting over the excised portions. An early return to full hand function is especially crucial in this patient, a laborer who has sustained a burn to his dominant right hand. Occupational therapy can be initiated immediately after excision and skin grafting to maximize hand function, allowing him to return to work as soon as possible.

Synthetic dressings should not be used to cover deep partial-thickness burns. Conservative therapy will only delay the appropriate treatment and make hand therapy more difficult. Coverage with a groin flap is an excessive, unnecessary procedure in this patient. Skin grafting should be delayed only if the depth and extent of the patient’s burn injuries are unknown. In these situations, a waiting period of seven days may be beneficial.

106
Q

A 47-year-old man sustains a high-voltage electrical burn injury to the left upper extremity; a photograph is shown above. Physical examination shows swelling and tenseness of the forearm; there is no circumferential eschar. An exit wound is noted on the left foot. Adequate fluid resuscitation has been performed, and the patient is stable.

Which of the following is the most appropriate immediate management of the right forearm and hand?

(A) Observation
(B) Splinting
(C) Escharotomy
(D) Fasciotomy
(E) Amputation

A

The correct response is Option D.

Electrical injuries involving a charge of greater than 1000 volts are often misleading because the mildness of the superficial wounds frequently masks the serious underlying problems. Bone and muscle have greater resistance and thus generate significant heat, and the necrotic processes of these tissues are often hidden under viable skin. Serial excision of tissue is required to address this complication. Affected patients also have marked edema and rapidly increasing compartment pressures, which inhibit vascular inflow and can further worsen tissue necrosis. Muscle necrosis may lead to myoglobinuria, which if left untreated can result in myoglobin-induced renal failure. To prevent this life-threatening complication, the urine must be alkalized with administration of sodium bicarbonate, and urinary output must remain at a constantly high rate. Fasciotomies of the hand and forearm should be performed immediately for tissue salvage; the surgeon should continue to assess the viability of the tissue in the hand and forearm following fasciotomy.

Observation is obviously inadequate and even dangerous in a burn patient with compartment syndrome. Similarly, splinting alone will not prevent further injury to the extremity. Although escharotomy does not relieve compartment syndrome, it is a recommended first step in patients who have burn injuries with constricting eschar. Amputation prior to complete demarcation may be required in patients with infected or completely necrotic tissue.

107
Q

A 2-year-old child has the findings shown in the photograph above six days after sustaining a full-thickness burn injury to the left hand when the hand was immersed in boiling water. Silver sulfadiazine dressings have been applied since the time of injury. Which of the following is the most appropriate next step in management?

(A) Continued use of silver sulfadiazine dressings for three weeks
(B) Tangential excision and coverage with split-thickness skin grafts
(C) Tangential excision and coverage with full-thickness skin grafts
(D) Fascial excision and coverage with full-thickness skin grafts
(E) Coverage with cryopreserved acellular dermal homograft and epidermal grafts

A

The correct response is Option B.

In this 2-year-old child who has a full-thickness burn, the most appropriate management is tangential excision to a level at which punctate bleeding occurs, followed by split-thickness skin grafting. The depth of the burn can be determined by serial examination; in this case, after six days, the wound is not vascularized and has not become epithelized. Because burns that have not healed 21 days after initial injury are associated with a significant risk for hypertrophic scarring and contracture, this child’s burn should be excised tangentially to the level of punctate bleeding. In addition, early skin grafting decreases the risk for scarring and permanent stiffness in patients with burns of the hand and increases the rehabilitation potential.

As implied above, continued application of silver sulfadiazine dressings is not appropriate in this patient. Any available full-thickness skin for grafting would most likely not be sufficient for coverage of this full-thickness burn involving most of the hand and forearm; a full-thickness graft is recommended instead for smaller areas that will contract only minimally. Although acellular dermal homograft has been shown to be beneficial in larger burns, its effects are limited in patients with burns limited to the hand who have other donor sites available.

108
Q

Inadequate fluid resuscitation in a burn patient is most likely to result in which of the following conversion mechanisms?

(A) Zone of coagulation to zone of hyperemia
(B) Zone of coagulation to zone of stasis
(C) Zone of stasis to zone of coagulation
(D) Zone of stasis to zone of hyperemia

A

The correct response is Option C.

There are three zones of injury in burn patients. The zone of coagulation lies in the center of the wound and is characterized by irreversible tissue destruction. It is immediately surrounded by the zone of stasis, an area of decreased perfusion with demonstrated damage to the microvasculature. The surrounding zone of hyperemia sustains the least amount of damage.

During the first 48 hours following burn injury, patients who are not properly resuscitated are at increased risk for conversion of the zone of stasis to a zone of coagulation. In contrast, if appropriate resuscitation is begun immediately, the zone of stasis can be reversed, potentially preventing the development of necrosis.

109
Q

A 63-year-old man who was in a house fire has burns on 55% of his body including the upper limbs, chest, abdomen, and left leg. He underwent escharotomies and has been resuscitated, but he requires mechanical ventilation because of an inhalation injury. He has an evolving acute kidney injury. A photograph is shown. Immediate excision is planned for management of a suspected fungal infection of the burn wounds. Which of the following is the most appropriate method for initial excision in this patient?

A) Excision down to fascia without a tourniquet
B) Excision down to viable tissue using tumescence
C) Excision down to viable tissue with a tourniquet
D) Excision down to viable tissue without a tourniquet
E) Hydrosurgical debridement without a tourniquet

A

The correct response is Option A.

The best method for initial excision in this critically ill patient is excision down to fascia (fascial excision). Excision down to viable tissue (tangential excision) with or without a tourniquet would result in a large amount of blood loss, which would be a significant physiologic insult for this patient; additionally, it may not eradicate the suspected fungal infection. Tangential excision using tumescence may not result in significant blood loss, but the ability to judge viable from nonviable tissue is compromised, and this approach would have a high likelihood of requiring further debridement to achieve a healthy tissue bed unless performed by extremely experienced burn surgeons. Fascial excision is also much faster than tangential excision, which is an important consideration in this critically ill patient. Hydrosurgical debridement is adequate for superficial burns but has no role in a large flame burn and would result in excessive blood loss and operative time.

110
Q

A 33-year-old man who weighs 80 kg is brought to the emergency department eight hours after sustaining deep partial-thickness burns involving 25% total body surface area (TBSA) and full-thickness burns involving 15% TBSA. According to the Parkland formula, how many milliliters (mL) of crystalloid should be administered for initial fluid resuscitation over the next eight hours?

(A) 3200
(B) 6400
(C) 9600
(D) 12,800
(E) 16,000

A

The correct response is Option C.

In a patient who has a burn injury that covers more than 20% of the total body surface area (TBSA), acute fluid resuscitation should be performed with administration of crystalloid during the initial 24 hours after injury. The Parkland formula is used to estimate the amount of fluid required. According to this formula, lactated Ringer’s solution 4 mL/kg/% TBSA burned should be administered during the first 24 hours. A total of 50% of the solution should be administered during the first eight-hour period and the remaining 50% over the next 16 hours.

An 80-kg patient who has burns involving 40% TBSA will require 12,800 mL of fluid during the first 24 hours: 6400 mL during the first eight hours and the remaining 3200 mL in both the second and third eight-hour periods. Because he received no fluid during the first eight hours immediately following injury, 9600 mL of crystalloid should be administered over the next eight hours in order to adequately resuscitate the patient.

111
Q

In a patient who sustained third-degree burns one hour ago, which of the following mechanisms associated with the initial inflammatory response is most likely to result in progressive tissue destruction?

(A) Downregulation of integrins
(B) Increased chemotaxis
(C) Inhibition of neutrophil degranulation
(D) Inhibition of tumor necrosis factor-alpha and interleukins 1 and 8
(E) Replacement of neutrophils with macrophages

A

The correct response is Option B.

During the first few hours after burn injury, multiple cytokines act to mediate a massive inflammatory response. Tumor necrosis factor-alpha (TNF-_) and interleukins 1 and 8 are released, resulting in increased chemotaxis of neutrophils into the wound. Upregulation of integrins also occurs following the release of TNF-_ and interleukins, and cell surface adherence receptors appear on neutrophils and endothelial cells. During this phase, neutrophils migrating into the wound adhere to the capillary endothelium and degranulate, resulting in the release of proteases and toxic oxygen-free radicals, leading to further tissue destruction. Macrophages gradually replace neutrophils over the first few days following injury and produce cytokines, which are critical for wound healing.

112
Q

A 32-year-old man has severe pain and swelling of both hands after being exposed to hydrofluoric acid while working with a rust remover. On examination, there is significant edema, mottling, and exquisite tenderness of the index, long, and ring fingers of both hands. Digital pulses are present on Doppler ultrasonography.

Following copious irrigation of the hands with water, which of the following is the most appropriate next step in management?

(A) Irrigation with 1% copper sulfate
(B) Topical application of phenol
(C) Application of a calcium sulfate splint
(D) Local injection of 10% calcium gluconate
(E) Debridement of the wounds and coverage with split-thickness skin grafts

A

The correct response is Option D.

This patient has sustained chemical burns to both hands after coming in contact with hydrofluoric acid, a corrosive material derived from elemental fluoride and used in rust removal and plastic and pottery manufacturing. Hydrofluoric acid burns can result in necrosis of soft tissues and decalcification of bone; affected patients can have pain that persists for days. The mechanism of action of this type of burn is due to the high concentration of hydrogen ions within the tissues, as well as liquefaction necrosis caused by the soluble free fluoride ion. Following copious irrigation of the burn site with water to remove as much of the hydrogen ion as possible, 10% calcium gluconate should be injected locally in multiple small doses to prevent vascular compromise. The calcium will bind to the fluoride ion, resulting in immediate relief of pain. A topical calcium gluconate paste can be applied in patients who have less severe burns, and intra-arterial injection is advocated for patients with more severe burns.

Copper sulfate is used for irrigation in patients with phosphorus burns to identify buried particles of phosphorus. Phenol should not be applied because it can be absorbed through intact skin and further worsen injury. In patients who have sustained phenol burn injuries, topical application of polyethylene glycol or vegetable oil is recommended. Similarly, calcium sulfate (eg, plaster of Paris) can result in exothermic burns when used in a splint or cast. Debridement with split-thickness skin grafting should be considered only after the extent of demarcation of the injury is fully known; it may be necessary in patients who have persistent liquefaction necrosis, which manifests as unrelenting pain, even after treatment with calcium gluconate.

113
Q

A 2-year-old boy is brought to the emergency department after sustaining a burn to the corner of the mouth when he bit on an electric cord. A photograph is shown above. The most appropriate management is splinting of the oral commissure for a minimum of what period of time?

(A) Six weeks
(B) Two to three months
(C) Six months
(D) One year

A

The correct response is Option C.

In this child who has sustained a burn to the oral commissure, the oral commissure should be splinted for a period of at least six months. Approximately 90% of all burns of the oral commissure occur in children younger than age 4 years, and boys are twice as likely to be injured; most of these injuries occur when a child places a live electric cord into the mouth. The flow of saliva produces an electrical short, resulting in thermal damage to the tissues.

Because long-term splinting has been shown to prevent microstomia and preserve function in patients who have injuries confined to the oral commissure, a custom-made device (either fixed or removable) should be applied to compress the commissure. It should be worn continuously for a minimum of six months and then at night only for several more months.

In these children, there is a 10% incidence of bleeding from the labial artery following injury. The child’s parent should be instructed to place the thumb and finger on the artery to control the bleeding if this does occur.

In children who have more severe injuries or for whom splinting is not practical, early surgical intervention or delayed reconstruction following scar maturation is recommended. If the injury extends beyond the oral commissure, functional lip reconstruction should be performed.

114
Q

Eight months after sustaining a deep second-degree burn of the dorsal aspect of the right hand, a 45-year-old woman has hyperextension of the metacarpophalangeal joint of the little finger resulting from a progressively worsening scar contracture. A photograph is shown above. Intensive occupational therapy has not improved this patient’s condition. Following release of the scar contracture, which of the following is the most appropriate operative management?

(A) Thin split-thickness skin grafting
(B) Full-thickness skin grafting
(C) Cultured epithelial autografting
(D) Coverage with a free lateral arm flap
(E) Coverage with a radial forearm flap

A

The correct response is Option B.

This patient has a significant scar contracture after sustaining a deep second-degree burn, which by nature is defined as a partial-thickness burn. This type of burn is often associated with hyperextension scarring and preservation of the underlying extensor tendon mechanism. Following release of the scar contracture, this patient should undergo full-thickness skin grafting of the hand. The paratenon of the extensor mechanism will readily accept a full-thickness skin graft, which will maximize long-term mobility of the metacarpophalangeal joint. In addition, full-thickness skin
Both thin and thick split-thickness skin grafts have higher secondary contraction rates than full-thickness skin grafts, with thin split-thickness skin grafts having the greatest rate of secondary contraction.

Because full-thickness skin grafting will provide the best thin coverage of the burn wound, more difficult and sophisticated procedures, such as autografting or coverage with free or pedicled flaps, are unnecessary.

115
Q

Which of the following immunologic responses is most likely to be seen in a 50-year-old woman who has sustained a 50% total body surface area (TBSA) burn in a house fire?

(A) Augmented B lymphocyte function
(B) Decreased fibronectin levels
(C) Decreased quantity of suppressor T lymphocytes
(D) Increased complement activation
(E) Increased production of IgG and IgM antibodies

A

The correct response is Option B.
Immunologic responses anticipated in this 50-year-old woman who has sustained a 50% TBSA burn include decreased levels of fibronectin, diminished complement activation, and decreased production of immunoglobulin antibodies. Patients who sustain burn injuries enter into an immunocompromised state, in which the ability to perform the functions of phagocytosis and pathogen elimination are severely limited, resulting in an inability to produce fibronectin. In addition, there is a generalized depression of the cellular immune response, including a decrease in the quantity and function of both B and T lymphocytes; however, the number of suppressor T lymphocytes is actually increased following acute thermal injury.

116
Q

Which of the following physiologic mechanisms is increased during the first 24 hours following thermal burn injury?

(A) Cardiac output
(B) Central venous pressure
(C) Circulating erythrocyte volume
(D) Circulating glucose concentration
(E) Plasma volume

A

The correct response is Option D.

The circulating glucose concentration is increased during the first 24 hours following thermal burn injury. The affected patient develops glucose intolerance due to the release of catecholamines from the burn site. Because of this, glucose should not added to the fluids given intravenously for acute resuscitation.

Following burn injury, the release of myocardial depressants diminishes cardiac output. Cardiac output is decreased to 40% to 60% of normal as a result of decreased plasma volume and increased systemic vascular resistance. Cardiac output then returns to normal but is not increased. The aforementioned decrease in plasma volume, which occurs in part from a capillary leak, subsequently leads to a decrease in central venous pressure. In addition, there is a decrease in circulating erythrocyte volume, due in part to a direct destruction of erythrocytes by the injured tissue.

117
Q

In a patient who has an acute deep partial-thickness burn of the ear, which of the following is the most appropriate immediate management?

(A) Application of silver nitrate soaks
(B) Application of mafenide acetate dressings
(C) Application of silver sulfadiazine dressings
(D) Debridement of the wound and splinting
(E) Excision of the burn eschar and grafting

A

The correct response is Option B.

In a patient who has an acute deep partial-thickness burn of the ear, the most appropriate management is application of mafenide acetate dressings every 12 hours. This topical antimicrobial therapy will prevent the development of suppurative chondritis, a painful condition that develops within the auricular cartilage in patients with this type of burn. It is first seen three to five weeks after initial injury and is extremely difficult to treat once it is acquired. Because mafenide acetate has a broad antibacterial spectrum and will easily penetrate ear cartilage, it is advocated as initial treatment.

Silver nitrate has a broad antimicrobial spectrum but penetrates the burn wound only minimally. Although silver sulfadiazine is most commonly used for topical therapy in patients with burn wounds, it contains lower antibacterial levels than mafenide acetate and will not penetrate burn eschar. Avoiding aggressive debridement and extreme pressure on the affected ear (ie, the use of excessive dressings or a pillow) has been shown to markedly decrease the incidence of chondritis. Early excision and grafting are not recommended.

118
Q

A 25-year-old man has burn alopecia after sustaining a burn wound involving 35% of the hair-bearing scalp. Which of the following is the most appropriate method of reconstruction in this patient?

(A) Free flap reconstruction
(B) Micrografting
(C) Minigrafting
(D) Strip grafting
(E) Tissue expansion

A

The correct response is Option E.

In patients with burn alopecia, the hair-bearing area of the scalp is amenable to tissue expansion; therefore, it is most appropriate for reconstruction in this patient. With this technique, large areas of the scalp can be resurfaced with similar tissue, resulting in reliable, consistent hair growth.

Grafting techniques, such as the use of micrografts, minigrafts, or strip grafts, are appropriate for management of male pattern alopecia but are unreliable in a patient with a compromised recipient site, such as a burn wound. Free flap reconstruction will allow rapid wound healing in a patient who has acute extensive and/or deep injuries involving exposed, devitalized skull.

Although hair transplantation is an option in patients with burn-related alopecia, it would not be the treatment of choice in this patient because of the diminished vascularity and severe scarring seen in the recipient bed.

119
Q
A